Resp2 Exam2 PQs

Réussis tes devoirs et examens dès maintenant avec Quizwiz!

A 53-year-old woman with a history of cigarette smoking pres- ents with a 3-month history of chest pain, cough, and mild fever. A chest X-ray reveals a peripheral mass in the left upper lobe. The surgical specimen is shown in the image. What is the most likely diagnosis? (A) Adenocarcinoma (B) Large cell carcinoma (C) Mesothelioma (D) Small cell carcinoma (E) Squamous cell carcinoma

(A) Adenocarcinoma Adenocarcinoma usually presents as a peripheral subpleural mass composed of neoplastic gland-like structures. Central (hilar) cancers of the lung can be of any of the histologic types (e.g., choices B, D, and E), whereas peripheral lung cancers are most commonly diagnosed as adenocarcinomas. They are often associated with pleural fibrosis and subpleural scars. At initial presentation, adenocarcinomas usually appear as irregular masses, although they may be so large that they completely replace the entire lobe of the lung. Mesothelioma (choice C) is pleural based. Diagnosis: Adenocarcinoma of lung

A 58-year-old man presents with a long history of persis- tent cough, chest pain, and recurrent pneumonia. He denies smoking or consuming alcohol. The patient subsequently dies of sepsis. Autopsy reveals malignant cells that diffusely infiltrate the lung parenchyma. Histopathologic examina- tion of the lung shows well-differentiated, mucus-producing, columnar neoplastic cells lining the alveolar spaces (shown in the image). Neoplastic cells are not found in any other organ. What is the most likely diagnosis? (A) Bronchioloalveolar carcinoma (B) Carcinoid tumor (C) Large cell carcinoma (D) Mesothelioma (E) Small cell carcinoma

(A) Bronchioloalveolar carcinoma Bronchioloalveolar carcinoma is a primary pulmonary adenocarcinoma originating from stem cells in the terminal bronchioles. The cells may be columnar and mucus producing or cuboidal and similar to type II pneumocytes. They tend to grow along the alveolar septa, as depicted in the photomicrograph. A similar growth pattern may be seen in metastatic adenocarcinomas. None of the other tumors produce alveolar mucus or display alveolar spaces lined by a columnar epithelium. Diagnosis: Bronchioloalveolar carcinoma

A 22-year-old man presents with a 6-month history of increasing shortness of breath and persistent cough with rusty sputum. A chest X-ray shows diffuse bilateral alveolar infiltrates. Urine dipstick analysis reveals 2+ hematuria. A transbronchial lung biopsy is shown in the image. Linear deposits of IgG and complement C3 are detected in the alveolar basement membrane by immunofluorescence. Which of the following is the most likely diagnosis? (A) Churg-Strausssyndrome (B) Goodpasture syndrome (C) Hypersensitivity pneumonitis (D) Loefflersyndrome (E) Wegener granulomatosis

(B) Goodpasture syndrome Goodpasture syndrome is an autoimmune disease in which autoantibodies bind to the noncollagenous domain of type IV collagen. This connective tissue protein is a major structural component of both pulmonary and glomerular basement membranes. Local complement activation results in the recruitment of neutrophils, tissue injury, pulmonary hemorrhage, and glomerulonephritis. Anti-type IV collagen antibodies are not encountered in the other choices. Diagnosis: Goodpasture syndrome

A 31-year-old woman smoker complains of nonproductive cough, chest pain, shortness of breath on exertion, and fatigue. A CBC is normal. A chest X-ray shows ill-defined nodules, reticulonodular infiltrates, a small cavitary lesion in the right middle lobe, and mediastinal adenopathy. A transbronchial biopsy is shown in the image. Which of the following is the most likely diagnosis? (A) Goodpasturesyndrome (B) Langerhans cell histiocytosis (C) Lymphangioleiomyomatosis (D) Pulmonary interstitial fibrosis (E) Wegener granulomatosis

(B) Langerhans cell histiocytosis Different presentations of Langerhans cell histiocytosis have been called eosinophilic granuloma, Hand-Schuller-Christian disease, and Letterer-Siwe disease. In adults, the disorder occurs most often as an isolated form known as pulmonary eosinophilic granuloma. Virtually all of these patients are cigarette smokers. The pulmonary lesions consist of varying proportions of Langerhans cells admixed with lymphocytes, eosinophils, and macrophages. Eosinophils are not typical of the other choices. Diagnosis: Langerhans histiocytosis

A 55-year-old man presents with increasing chest pain, bloody sputum, and weight loss over the past 3 months. A high-resolution CT scan reveals a mass circumscribing the right main bronchus, extending into its lumen. Histologic examination of an open-lung biopsy is shown in the image. Electron microscopy shows numerous neuroendocrine granules within tumor cells. What is the appropriate diagnosis? (A) Adenocarcinoma (B) Bronchioloalveolar carcinoma (C) Carcinoid tumor (D) Large cell carcinoma (E) Squamous cell carcinoma

(C) Carcinoid tumor Carcinoid tumors account for 2% of all primary lung cancers. They comprise a group of neuroendocrine neoplasms derived from the pluripotential basal layer of the respiratory epithelium. Carcinoid tumors occur most often in the wall of the major bronchus and may protrude into its lumen. The tumors are characterized by an organoid growth pattern and uniform cytologic features. Carcinoid tumors exhibit a neuroendocrine differentiation similar to that of resident Kulchitsky cells. The indolent nature of carcinoid tumors is reflected in the finding that half of the patients are asymptomatic at the time of presentation, but regional lymph node metastases occur in 20% of patients. Atypical carcinoids exhibit a more aggressive behavior. Neuroendocrine features are absent in the other tumors. Diagnosis: Carcinoid tumor of lung

A 38-year-old woman, who is being treated with corticos- teroids for systemic lupus erythematosus, presents with chronic nonproductive cough. She breeds pigeons for avian hobbyists. A chest X-ray reveals a 2-cm nodule in the upper lobe of the right lung. The lung nodule is resected. Histo- logic examination reveals granulomas and budding yeast forms, which stain positively for polysaccharides (mucicarmine stain, shown in the image). What is the appropriate diagnosis? (A) Actinomycosis (B) Coccidioidomycosis (C) Cryptococcosis (D) Histoplasmosis (E) Mycoplasma pneumonia

(C) Cryptococcosis Cryptococcosis results from the inhalation of spores of Cryptococcus neoformans, an organism frequently encountered in pigeon droppings. Most serious cases occur in immunocompromised persons. Other examples of fungal infections of the lungs are histoplasmosis (choice D), coccidioidomycosis (choice B), and aspergillosis. However, cryptococcus stains positively with a mucicarmine stain for capsular polysaccharides. These diseases are also acquired by inhaling spores. Diagnosis: Cryptococcal pneumonia, pigeon breeder lung disease

A 53-year-old man develops weakness, malaise, cough with bloody sputum, and night sweats. A chest X-ray reveals numerous apical densities bilaterally, some of which are cavi- tary. Exposure to Mycobacterium tuberculosis was documented 20 years ago, and M. tuberculosis is identified in his sputum. Which of the following describes the expected lung pathology in this patient? (A) Densefibrosis (B) Eosinophilic infiltration (C) Granulomas (D) Interstitialpneumonia (E) Plasma cell infiltration

(C) Granulomas Secondary (reactivation) tuberculosis is characterized by the formation of granulomas and extensive tissue destruction (caseous necrosis). Mycobacteria typically spread to the apices of the lungs and produce large cavities, which are associated with hemoptysis. Miliary tuberculosis refers to widespread seeding of bacteria in the lungs and distant organs. Granulomatous inflammation may induce fibrosis (choice A) as a secondary feature. Diagnosis: Tuberculosis, Mycobacterium tuberculosis

A 67-year-old woman with a history of smoking presents with a 3-week history of chest pain and bloody sputum. A chest X-ray reveals a bulky mass within the pulmonary parenchyma. An open-lung biopsy is shown in the image. Immunohistochemical stains for keratin and chromogranin are negative. What is the appropriate diagnosis? (A) Adenocarcinoma (B) Carcinoid tumor (C) Large cell carcinoma (D) Metastaticadenocarcinoma (E) Small cell carcinoma

(C) Large cell carcinoma Large cell undifferentiated carcinoma is composed of atypical neoplastic cells that do not resemble any normal cells in the lung. These cells do not form glands (like adenocarcinoma) and do not express cytokeratin (choices A and D). Chromogranin is expressed in carcinoid tumors (choice B) and often in small cell carcinomas (choice E). Diagnosis: Large cell carcinoma of lung

A 48-year-old man with AIDS is admitted to the hospital with a fever of 38.7°C (103°F). The patient has a 2-week history of persistent cough and diarrhea. Laboratory studies show that the CD4+ cell count is less than 500/μL. A sputum culture reveals acid-fast organisms, which are further identified as Mycobacterium avium-intracellulare. This patient's pneumonia is characterized by extensive pulmonary infiltrates of which of the following cell types? (A) CD4+ helper T cells (B) Eosinophils (C) Macrophages (D) Mastcells (E) Neutrophils

(C) Macrophages Mycobacterium avium intracellulare (MAI) complex is a progressive systemic disorder, often occurring in patients who have AIDS. 1/3 of all patients with AIDS develop overt MAI infections because depletion of CD4+ helper T cells cripples the immune response. This pneumonia is characterized by an extensive infiltrate of macrophages. The proliferation of MAI and the recruitment of macrophages produce expanding lesions, ranging from epithelioid granulomas containing few organisms to loose aggregates of foamy macrophages. Symptoms associated with MAI resemble those of tuberculosis. The other inflammatory cells listed are not characteristic of infection with MAI. Diagnosis: Mycobacterium avium-intracellulare, AIDS

A 40-year-old woman with leukemia is treated with chemo- therapy. During treatment she develops increasing cough and shortness of breath. A chest X-ray shows diffuse lung infil- trates. Sputum cultures are negative, and the patient does not respond to routine antibiotic therapy. An open lung biopsy is diagnosed by the pathologist as viral pneumonia. Which of the following histopathologic findings would be expected in the lungs of this patient? (A) Clusters of epithelioid macrophages (B) Confluent areas of caseous necrosis (C) Fibrous scarring of lung parenchyma (D) Hyaline membranes and interstitial inflammation (E) Sheets of bacilli-filled macrophages

(D) Hyaline membranes and interstitial inflammation Viral infections of the pulmonary parenchyma produce diffuse alveolar damage (DAD) and interstitial pneumonia. Necrosis of type I pneumocytes and the formation of hyaline membranes result in an appearance that is indistinguishable from DAD in other settings. Choices A, B, and D are not characteristic of interstitial pneumonia. Choice C (fibrous scarring) may be a late complication of some forms of this disorder. Diagnosis: Diffuse alveolar damage, viral pneumonia

A 40-year-old alcoholic man is admitted to the hospital in severe respiratory distress. The temperature is 38.7°C (103°F), respirations are 32 per minute, and blood pressure is 130/90mmHg. He coughs constantly and expectorates "currant-jelly" sputum. A chest X-ray reveals bilateral diffuse pulmonary consolidation. Physical examination shows bilat- eral crackles, dullness to percussion over both pulmonary fields, and use of accessory muscles. The patient subsequently dies from complications of bacterial sepsis. The left lung at autopsy (shown in the image) shows a red, engorged lower lobe. What is the appropriate diagnosis? (A) Atypical pneumonia (B) Bronchopneumonia (C) Interstitial pneumonia (D) Lobar pneumonia (E) Pulmonary abscess

(D) Lobar pneumonia The term lobar pneumonia refers to consolidation of an entire lobe; bronchopneumonia (choice B) signifies scattered solid foci in the same or several lobes. Lobar pneumonia presents with a diffuse consolidation of one or more pulmonary lobes. In contrast to lobar pneumonia, interstitial pneumonia (choice C) primarily involves the alveolar septa. Atypical pneumonia (choice A) is most often encountered in mycoplasma pneumonia. Pulmonary abscess (choice E) may be a complication of lobar pneumonia or bronchopneumonia. Diagnosis: Lobar pneumonia

A 65-year-old man who is a heavy smoker complains of sud- den onset of malaise, fever, productive cough, abdominal pain, and muscle aches. A chest X-ray shows bilateral, diffuse, patchy, alveolar infiltrates. Laboratory studies reveal that the patient is infected with Legionella pneumophila. This patient's pneumonia is characterized by extensive pulmonary infiltrates of which of the following cell types? (A) CD4+ helper T cells (B) CD8+ killer T cells (C) Eosinophils (D) Macrophages (E) Mast cells

(D) Macrophages Legionella pneumonia begins when microorganisms enter the alveoli, where they are phagocytozed by macrophages. Bacteria multiply within macrophages and are released to infect new macrophages. Smoking, alcoholism, and chronic pulmonary diseases interfere with normal host defenses thereby increasing the risk of developing Legionella pneumonia. Patients typically present with an acute bronchopneumonia. 1/3 of cases of Legionella pneumonia are complicated by subsequent emphysema. The other inflammatory cells listed are scarce or absent in the alveolar exudate. Diagnosis: Legionnaire disease

A 64-year-old man who has worked in a manufacturing plant all his life complains of an 8-month history of chest discom- fort, malaise, fever, night sweats, and weight loss. A chest X-ray reveals a pleural effusion and pleural mass encasing the lung. The patient subsequently dies of cardiorespiratory failure. Histologic examination of the pleural mass at autopsy shows a biphasic pattern of epithelial and sarcomatous ele- ments. What is the most likely diagnosis? (A) Carcinoidtumor (B) Large cell carcinoma (C) Malignant melanoma (D) Malignant mesothelioma (E) Metastatic carcinoma

(D) Malignant mesothelioma Mesothelioma is a malignant neoplasm of mesothelial cells that is most common in the pleura, but also occurs in the peritoneum, pericardium, and the tunica vaginalis of the testis. The tumor is strongly linked to occupational inhalation of asbestos. Patients are often first seen with a pleural effusion or a pleural mass, chest pain, and nonspecific symptoms, such as weight loss and malaise. Pleural mesotheliomas tend to spread locally and extensively within the chest cavity, but do not typically invade the pulmonary parenchyma. Widespread metastases can occur. Mesothelioma is typically composed of both epithelial and sarcomatous elements (i.e., biphasic pattern). The other choices do not ordinarily encase the lung. Diagnosis: Malignant mesothelioma

If the patient described in Question 2 is appropriately treated with antibiotics, which of the following is the most likely outcome? (A) Abscessformation (B) Bronchopleural fistula (C) Bullousemphysema (D) Resolution (E) Scar formation

(D) Resolution Although abscesses and fistulasmay occur (choices A and B), the most common outcome of acute bacterial pneumonia is resolution, particularly with appropriate antibiotic treatment. Diagnosis: Pneumonia, Streptococcus pneumoniae

A 56-year-old man with a history of cigarette smoking pres- ents with difficulty swallowing and a muffled voice. Laryngoscopy reveals a 2-cm laryngeal mass. If this mass is a malignant neoplasm, which of the following is the most likely histologic diagnosis? (A) Adenocarcinoma (B) Leiomyosarcoma (C) Small cell carcinoma (D) Squamous cell carcinoma (E) Transitional cell carcinoma

(D) Squamous cell carcinoma The vast majority of laryngeal cancers are squamous cell carcinomas and occur principally in smokers. Adenocarcinoma (choice A), leiomyosarcoma (choice B), and small cell carcinoma (choice C) are rarely encountered in the larynx. Diagnosis: Laryngeal cancer

A 10-year-old boy suffers head trauma and lies unconscious for 2 weeks. He is now intubated. His temperature rises to 38.7°C (103°F), and oxygenation becomes more diffi- cult. A chest X-ray reveals a pleural effusion and multiple abscesses in the lung parenchyma. Which of the following microorganisms is the most likely cause of this pulmonary infection? (A) Legionella pneumophila (B) Mycoplasma pneumoniae (C) Pneumocystis carinii (D) Staphylococcus aureus (E) Streptococcus pneumoniae

(D) Staphylococcus aureus Staphylococcal pneumonia is an uncommon community-acquired disease, accounting for only 1% of bacterial pneumonias. However, pulmonary infection with Staphylococcus aureus is common as a superinfection after influenza and other viral respiratory tract infections. Nosocomial (hospital acquired) staphylococcal pneumonia typically occurs in chronically ill patients who are prone to aspiration or who are intubated. Although lung abscess can conceivably follow any respiratory infection, the other choices do not usually do so. Diagnosis: Bacterial pneumonia

A 22-year-old man with AIDS complains of persistent cough, night sweats, low-grade fever, and general malaise. A chest X-ray reveals an area of consolidation in the periphery of the left upper lobe, as well as hilar lymphadenopathy. Sputum cultures show acid-fast bacilli. Which of the following is the most likely diagnosis? (A) Bronchopneumonia (B) Pulmonary abscess (C) Sarcoidosis (D) Tuberculosis (E) Wegener granulomatosis

(D) Tuberculosis Tuberculosis represents infection with Mycobacterium tuberculosis, although atypical mycobacterial infections may mimic it. The Ghon complex includes parenchymal consolidation and enlargement of ipsilateral hilar lymph nodes and is often accompanied by a pleural effusion. The sputum contains M. tuberculosis, which is acid-fast in smears stained by the Ziehl-Neelsen technique. After resolution of primary tuberculosis, reemergence may occur (secondary tuberculosis). None of the other choices feature acid-fast organisms. Diagnosis: Tuberculosis, Mycobacterium tuberculosis

A 56-year-old woman with disseminated breast cancer undergoes multidrug chemotherapy. Ten days later, she develops cough and a fever of 38.7°C (103°F). A chest X-ray shows multiple areas of consolidation and a large cavity in the right upper lobe. Multiple pulmonary infarcts are also identified. The patient subsequently dies of multisystem organ failure. Histologic examination of the lungs at autopsy is shown in the image. Which of the following is the most likely diagnosis? (A) Actinomycosis (B) Blastomycosis (C) Coccidioidomycosis (D) Cryptococcosis (E) Invasive aspergillosis

(E) Invasive aspergillosis Invasive aspergillosis is the most serious manifestation of Aspergillus infection, occurring almost exclusively as an opportunistic infection inimmunocompromised persons (e.g., undergoing cytotoxic therapy or diagnosed with AIDS). Invasion of blood vessels and tissue infarcts are common. Aspergillus species may also grow in preexisting cavities caused by tuberculosis or bronchiectasis. They proliferate to form fungus balls, which are also referred to as aspergillomas or mycetomas. Vascular invasion is not a feature of the other choices. Diagnosis: Pulmonary aspergillosis

A 43-year-old woman with Sjögren syndrome and a 5-year history of cough and shortness of breath develops end-stage lung disease and dies of respiratory failure. Histologic exami- nation of the lung at autopsy is shown in the image. Which of the following is the most likely diagnosis? (A) Alveolar proteinosis (B) Churg-Strauss syndrome (C) Langerhans cell histiocytosis (D) Lymphangioleiomyomatosis (E) Lymphocytic interstitial pneumonia

(E) Lymphocytic interstitial pneumonia Lymphocytic interstitial pneumonia (LIP) is a rare pneumonitis in which lymphoid infiltrates are distributed diffusely in the interstitial spaces of the lung. In this case, the walls of the alveolar septa are diffusely infiltrated with chronic inflammatory cells. LIP often occurs in a variety of clinical settings, including Sjögren syndrome and HIV infection. The course of the disease varies from an indolent condition to one that progresses to end-stage lung disease and respiratory failure. Langerhans cell histiocytosis (choice C) features nodular infiltrates. Interstitial lymphocytic infiltrates are not characteristic of the other choices. Diagnosis: Lymphocytic interstitial pneumonia

A 48-year-old woman with a long-standing history of ulcer- ative colitis presents with anemia and shortness of breath. Laboratory studies show increased serum levels of carcinoembryonic antigen. A chest X-ray reveals multiple, round masses in both lungs. Histologic examination of an open-lung biopsy discloses nodules that are composed of gland-like structures. What is the most likely diagnosis? (A) Adenocarcinoma (B) Bronchioloalveolar carcinoma (C) Eosinophilic granuloma (D) Large cell undifferentiated carcinoma (E) Metastatic carcinoma

(E) Metastatic carcinoma Pulmonary metastases represent the most common neoplasm of the lung. In one third of all fatal cancers, pulmonary metastases are evident at autopsy. Metastatic carcinomas typically present as multiple, round masses scattered at random throughout the parenchyma of lungs and liver. Although pulmonary adenocarcinoma (choice A) and bronchoalveolar carcinoma (choice B) cannot be excluded on histologic grounds, this patient with ulcerative colitis is predisposed to develop adenocarcinoma of the colon, which most likely accounts for the anemia and lung metastases. Diagnosis: Metastatic carcinoma of lung

A 60-year-old alcoholic woman presents to the emergency room with fever, chills, and shortness of breath. The sputum is rusty-yellow and contains numerous neutrophils, red blood cells, and Gram-positive cocci. A chest X-ray shows diffuse haziness over both lungs. One week following admission, the patient develops empyema. This pulmonary condition is asso- ciated with the spread of bacterial infection to which of the following anatomic locations? (A) Blood (B) Bronchi (C) Interstitialspace (D) Pericardium (E) Pleural space

(E) Pleural space Complications of bacterial pneumonia include pleuritis (extension of inflammation to the pleural surface), pleural effusion, pyothorax (infection of pleural effusion), pulmonary abscess, and pulmonary fibrosis. Empyema is a loculated collection of pus with fibrous walls that follows the spread of bacterial infection to the pleural space. All of the other choices are possible routes of spread, but do not describe empyema. Diagnosis: Bacterial pneumonia, empyema

A 36-year-old man with AIDS presents with fever, dry cough, and dyspnea. A chest X-ray shows bilateral and diffuse infiltrates. Laboratory studies reveal a CD4+ cell count of less than 50/μL. A lung biopsy discloses a chronic interstitial pneumonitis and an intra-alveolar foamy exudate. A silver stain of a bronchoalveolar lavage is shown in the image. Which of the following organisms is the most likely pathogen responsible for these pulmonary findings? (A) Cryptococcus neoformans (B) Cytomegalovirus (C) Histoplasma capsulatum (D) Mycoplasma pneumoniae (E) Pneumocystis jiroveci

(E) Pneumocystis jiroveci P. jiroveci (formerly P. carinii ) is the most frequent cause of infectious pneumonia in patients with AIDS. Once considered a protozoan, the organism has been reclassified as a fungus. The classic lesion of Pneumocystis pneumonia comprises an interstitial infiltrate of plasma cells and lymphocytes, diffuse alveolar damage, and hyperplasia of type II pneumocytes. The alveoli are filled with a characteristic foamy exudate. The organisms appear as small bubbles in a background of proteinaceous exudates. In this case, a centrifuged bronchoalveolar lavage specimen impregnated with silver shows a cluster of cysts. The cysts appear as round or indented ("crescent moon") bodies, which are approximately 5 μm in diameter. Cryptococcus neoformans (choice A) and Histoplasma capsulatum (choice C) do not typically cause interstitial pneumonia. Diagnosis: Pneumocystis pneumonia, AIDS

A 56-year-old man undergoes a routine chest radiograph as part of a comprehensive physical examination. The X-ray film of the chest shows a solitary, centrally located coin lesion, with a "popcorn" pattern of calcification. A lung biopsy is per- formed and reveals nodules of benign mature cartilage and respiratory epithelium (shown in the image). What is the most likely diagnosis? (A) Carcinoid tumor (B) Extralobar sequestration (C) Leiomyoma (D) Pulmonaryfibroma (E) Pulmonary hamartoma

(E) Pulmonary hamartoma Although the term hamartoma implies a malformation, hamartomas are true tumors. They are composed of cartilage, fibromyxoid connective tissue, fat, bone, and occasional smooth muscle. They typically occur in adults, with a peak in the sixth decade of life. Hamartomas are the cause of approximately 10% of "coin" lesions discovered incidentally on chest radiographs. A characteristic "popcorn" pattern of calcification is often seen by X-ray. Cartilage is not encountered in the other choices. Diagnosis: Pulmonary hamartoma

A 52-year-old woman presents with a 1-year history of upper truncal obesity and moderate depression. Physical examination shows hirsutism and moon facies. Endocrinologic studies reveal hypokalemia, high plasma corticotropin levels, and increased concentrations of serum and urine cortisol. CT scan of the thorax demonstrates a hilar mass. A transbronchial lung biopsy is shown in the image. Electron microscopy discloses neuroendocrine granules within the cytoplasm of some tumor cells. What is the appropriate diagnosis? (A) Adenocarcinoma (B) Bronchioloalveolar carcinoma (C) Carcinoidtumor (D) Metastaticcarcinoma (E) Small cell carcinoma

(E) Small cell carcinoma Small cell carcinoma (previously referred to as "oat-cell" carcinoma) is a highly malignant epithelial tumor of the lung that exhibits neuroendocrine features. It accounts for 20% of all lung cancers and is strongly associated with cigarette smoking. Metastases occur early and are widespread. Carcinoid tumors (choice C) also contain neuroendocrine granules, but the tumor cells are arranged in a distinctive pattern. Moreover, Cushing syndrome is often encountered in patients with small cell carcinoma, but not carcinoid tumor (choice C). Diagnosis: Small cell carcinoma of lung

A 68-year-old man complains of shortness of breath, hoarseness, productive cough, and bloody sputum of 2 weeks in duration. He admits to smoking two packs a day for 45 years and drinks occasionally. Recently, he has experienced a significant loss of appetite and weight loss. Physical examination shows pallor, cachexia, clubbing of the fingers, and barrel-shaped chest. A chest X-ray reveals a mass at the right lung apex. Histologic examination of a transbronchial biopsy is shown in the image. What is the appropriate histologic diagnosis? (A) Adenocarcinoma (B) Mesothelioma (C) Metastatic adenocarcinoma (D) Small cell carcinoma (E) Squamous cell carcinoma

(E) Squamous cell carcinoma Squamous cell carcinoma accounts for 30% of all invasive lung cancers in the United States. Well-differentiated squamous cell carcinoma displays keratin "pearls," which appear as a small round nest of brightly eosinophilic aggregates of keratin surrounded by concentric ("onion skin") layers of squamous cells. Gland formation is exhibited in adenocarcinoma (choices A and C). Diagnosis: Squamous cell carcinoma of lung

A 64-year-old man presents with fever, chills, and increasing shortness of breath. The patient appears in acute respiratory distress and complains of pleuritic chest pain. Physical exami- nation shows crackles and decreased breath sounds over both lung fields. The patient exhibits tachypnea, with flaring of the nares. The sputum is rusty-yellow and displays numerous neutrophils and erythrocytes. Which of the following patho- gens is the most common cause of this patient's pulmonary infection? (A) Legionella pneumophila (B) Mycoplasma pneumoniae (C) Pseudomonas aeruginosa (D) Staphylococcus aureus (E) Streptococcus pneumoniae

(E) Streptococcus pneumonia All of the choices cause pneumonia. However, despite the impact of antibiotic therapy, pneumonia caused by Streptococcus pneumonia (pneumococcus) remains the most significant problem. The onset of pneumococcal pneumonia is acute with fever and chills. Diagnosis: Pneumonia, Streptococcus pneumoniae

A 23-year-old man complains of nasal obstruction, sero- sanguinous discharge, cough, and bloody sputum. A chest X-ray shows cavitated lesions and multiple nodules over both lung fields. A CT scan discloses obliteration of several maxillary sinuses. Urinalysis reveals hematuria and RBC casts. Laboratory studies demonstrate anemia and elevated serum levels of C-ANCA. An open-lung biopsy is shown in the image. Which of the following is the most likely diagnosis? (A) Adenocarcinoma of lung (B) Churg-Strauss syndrome (C) Necrotizing sarcoid granulomatosis (D) Tuberculosis (E) Wegener granulomatosis

(E) Wegener granulomatosis WG is a disease of unknown cause that is characterized by aseptic, necrotizing, granulomatous inflammation and vasculitis. This disease affects the upper and lower respiratory tract and kidneys. Pulmonary features of WG include necrotizing granulomatous inflammation, parenchymal necrosis, and vasculitis. In most cases, multiple nodules averaging 2 to 3 cm in diameter are seen in the lungs. WG most commonly affects the head and neck, followed by the lung, kidney, and eye. Respiratory manifestations include sinusitis, cough, hemoptysis, and pleuritis. Sinus involvement is not common in the incorrect choices. Churn-Strauss syndrome (choice B) shares some features with WG, but is characterized by asthma, peripheral eosinophilia and P-ANCA. Diagnosis: Wegener granulomatosis

Three weeks after visiting her grandmother dying from a respiratory tract infection, a healthy 5-year-old girl develops a fever along with dyspnea. On physical examination her temperature is 37.9°C. Her lung fields are clear to auscultation but there are expiratory wheezes. A chest x-ray reveals a solitary 2 cm peripheral mid-lung nodule and marked hilar lymphadenopathy. Laboratory studies show Hgb 13.6 g/dL, platelet count 183,600/microliter, and WBC count 5480/microliter. These findings are most consistent with infection by which of the following organisms? A Mycobacterium tuberculosis B Candida albicans C Coccidioides immitis D Aspergillus flavus E Bacteroides fragilis F Streptococcus pneumoniae G Respiratory syncytial virus

A - Mycobacterium tuberculosis The pattern of lung involvement is the classical 'Ghon complex' of primary tuberculosis, which is seen more commonly in children, though only about 5% of cases are symptomatic. The enlarged hilar nodes can impinge upon central airways to produce obstruction. B - Candidiasis is seen in persons who are immunocompromised. A true Candida pneumonia is rare. Candida is more likely to colonize the upper respiratory tract. C - Fungal granulomatous disease can mimic tuberculosis but is less common overall and is more typically seen in adults. D - This is most likely to be seen in immunocompromised adults. E - Lung abscesses from B. fragilis are not common in healthy children. Abscesses usually complicate a previous pulmonary infection or occur with aspiration. F - Pneumococcal infections can produce a lobar pneumonia or bronchopneumonia. G - RSV can produce childhood pneumonias, but they are typically interstitial pneumonias.

A 44-year-old man, a nonsmoker, has experienced a 3-kg weight loss over the past 3 months. He recently devel- oped a low-grade fever and cough with mucoid sputum pro- duction, and after 1 week, he noticed blood-streaked sputum. On physical examination, his temperature is 37.7° C. There are bilateral crackles in the left upper lobe on auscultation of the chest. Chest CT scan shows a 3-cm left upper lobe nodule with decreased attenuation centrally. Laboratory studies show he- moglobin, 14.5 g/dL; platelet count, 211,400/mm3; and WBC count, 9890/mm3 with 40% segmented neutrophils, 2% bands, 40% lymphocytes, and 18% monocytes. Which of the follow- ing findings in his sputum sample is most likely to be present? A Acid-fast bacilli B Branching septate hyphae C Charcot-Leyden crystals D Foreign body giant cells E Gram-negative bacilli F Small dark neoplastic cells

A Acid-fast bacilli This "coin lesion" on imaging of his lungs could be an infectious granuloma, a neoplasm, or a hamartoma. His fever suggests infection, and the CT finding of decreased central attenuation in the nodule suggests necrosis in a neoplasm or caseous necrosis in a granuloma. The lymphocytosis and monocytosis are consistent with tuberculosis. Nonsmokers are unlikely to develop primary lung neoplasms, and adenocarcinoma is the most common in that setting. Smokers are most likely to develop squamous cell carcinomas and small cell anaplastic carcinomas. Individuals who are immunocompromised are most likely to develop fungal infections, particularly with Aspergillus spp., which have branching septate hyphae. Charcot-Leyden crystals form from eosinophil granules in individuals with allergic asthma. Foreign body giant cells can be seen with lipid pneumonias. Gram stain is most useful for determining which bacterial organisms may be present, and gram-negative bacilli such as the Enterobacteriaceae produce acute pneumonias and abscesses with neutrophilia.

A 60-year-old woman has had a chronic nonproductive cough for 4 months along with loss of appetite and a 6-kg weight loss. She does not smoke. On physical examination there are no remarkable findings. Her chest radiograph shows a right peripheral subpleural mass. A fine-needle aspiration biopsy is performed, and she undergoes a right lower lobectomy. The microscopic appearance of the lesion is shown in the figure. She receives immunotherapy directed at epithelial growth factor receptor (EGFR) and remains symptom-free for the next 10 years. Which of the following neoplasms did she most likely have? A Adenocarcinoma B Bronchial carcinoid C Hamartoma D Large cell carcinoma E Small cell anaplastic carcinoma F Squamous cell carcinoma

A Adenocarcinoma Cancers that arise in nonsmokers are pathogenically distinct from those that occur in smokers. They may have either EGFR mutations or KRAS mutations. Most are adenocarcinomas. Twenty-five percent of lung cancers worldwide occur in nonsmokers. Primary adenocarcinomas in the lung tend to be small, peripheral masses that are amenable to surgical excision and have a better overall prognosis than other forms of lung cancer. Overall, far more metastatic adenocarcinomas involve the lung than do primary adenocarcinomas. Bronchial carcinoids are uncommon endobronchial lesions. Hamartomas are small, peripheral masses that contain benign epithelial and connective tissue elements. Large cell carcinomas are too poorly differentiated to be called adenocarcinomas or squamous cell carcinomas. The most common cancers in smokers are small cell anaplastic and squamous cell carcinomas.

A 79-year-old woman has had increasing malaise and a 5-kg weight loss over the past 5 months. She has had a chronic nonproductive cough for 3 months. On physical examination, she has no abnormal findings. Pulmonary function tests are normal. Her peripheral blood counts are normal. Her chest radiograph is shown in the figure. What is a biopsy of one of her lung lesions most likely to show? A Adenocarcinoma B Granulomatous inflammation C Necrotizing vasculitis D Organizing abscess E Silica crystals

A Adenocarcinoma The long history and the weight loss suggest a chronic, debilitating process. The radiograph shows multiple mass lesions. The focal nature of the lesions, with remaining pulmonary reserve capacity, means that measured lung function remains normal. The most common neoplastic process of lung is metastatic disease, because many primary sites outside of lung can gain vascular access and spread hematogenously, and lung has a rich capillary bed for tumor cell emboli to colonize. Immunohistochemical markers on this adenocarcinoma might help characterize the primary site, or they might not. Depending upon the markers, there might be specific antineoplastic therapies, or not. The lack of fever and any evidence for inflammation makes an infectious process unlikely. Necrotizing vasculitis is likely to produce hemoptysis. Silicosis is unlikely without an environmental exposure in a job setting, and the lesions lead to restrictive lung disease.

A 57-year-old woman has had a cough and pleuritic chest pain for the past 3 weeks. On physical examination, she is afebrile. Some crackles are audible over the left lower lung on auscultation. A chest radiograph shows an ill-defined area of opacification in the left lower lobe. After 1 month of antibiotic therapy, her condition has not improved, and the lesion is still visible radiographically. CT-guided needle biopsy of the left lower lobe of the lung is performed, and the specimen has the histologic appearance shown in the figure. Which of the following neoplasms is most likely to be present in this patient? A Adenocarcinoma in situ B Large cell anaplastic carcinoma C Malignant mesothelioma D Metastatic breast carcinoma E Squamous cell carcinoma

A Adenocarcinoma in situ Adenocarcinoma in situ (AIS), formerly termed bronchioloalveolar carcinoma, can present as a peripheral tumor that can mimic pneumonia. Most of these tumors are well differentiated. Adenocarcinomas and large cell carcinomas tend to be peripheral, but the former tend to produce a localized mass, whereas cells of the latter are large and pleomorphic and form sheets; sometimes it is difficult to distinguish among them. Mesotheliomas almost always occur in the setting of prior asbestos exposure; they are large pleural masses. Metastases tend to appear as multiple nodules. Squamous cell carcinomas occasionally can be peripheral (although most are central) and are composed of pink, polygonal cells that have intercellular bridges. If well differentiated, squamous cell carcinomas show keratin pearls.

A 78-year-old man has had increasing dyspnea without cough or increased sputum production for the past 4 months. On physical examination, he is afebrile. Breath sounds are reduced in all lung fields. A chest CT scan shows a dense, brightly attenuated pleural mass encasing most of the left lung. Microscopic examination of a pleural biopsy specimen shows spindle and cuboidal cells that invade adipose tissue. Inhalation of which of the following pollutants is the most likely factor in the pathogenesis of this mass? A Asbestos B Bird dust C Coal dust D Cotton fibers E Ozone F Silica

A Asbestos Malignant mesothelioma is a rare tumor even in individuals with a history of asbestos exposure. The tumor may appear decades after exposure, and is not related to amount or length of exposure. Bronchogenic carcinoma is more common in individuals with asbestos exposure, particularly when there is a history of smoking. Bird dust inhalation can lead to hypersensitivity pneumonitis. Coal dust inhalation can lead to marked anthracosis, but without a significant risk of lung cancer. Inhalation of cotton fibers (byssinosis) leads to symptoms resembling asthma related to bronchoconstriction. Ozone and nitrogen oxides in smog can cause acute respiratory discomfort, but are not known to be promoters of neoplasia. Silicosis is typified by interstitial fibrosis and causes a slight increase in the risk of bronchogenic carcinoma.

A 56-year-old man is undergoing chemotherapy for leu- kemia. He has developed fever, nonproductive cough, dys- pnea, pleuritic chest pain, and hemoptysis over the past week. A chest CT scan shows multiple 1- to 4-cm nodular densities having surrounding areas of ground-glass infiltrate (halo sign). Bronchoalveolar lavage is performed, and microscopic exami- nation of the fluid shows narrow branching septate hyphae. A CBC shows Hgb, 13 g/dL; Hct, 38.7%; WBC count, 2000/μL; and platelet count, 200,100/μL. He has most likely developed an infection with which of the following organisms? A Aspergillus fumigatus B Candida albicans C Cryptococcus neoformans D Moraxella catarrhalis E Mucor circinelloides

A Aspergillus fumigatus There are several patterns of pulmonary involvement with Aspergillus spp. Immunocompromised patients with neutropenia may develop invasive aspergillosis. Other patterns include allergic bronchopulmonary aspergillosis in persons with asthma and an aspergilloma, or fungus ball, colonizing a cavitary lesion of tuberculosis or bronchiectasis. Candidiasis may also develop in the setting of neutropenia, but less commonly causes extensive lung involvement, appears as budding cells with pseudohyphae, and more likely produces an oral, nasal, or pharyngeal infection. Cryptococcosis can cause extensive pulmonary infections, particularly with loss of cell-mediated immunity, and the organisms have large mucoid capsules and exhibit narrow-based budding. Moraxella is a bacterial organism most often causing sinusitis, otitis, and upper respiratory infections. Mucor appears as broad, nonseptated hyphae and is most often a complication of diabetic ketoacidosis, with nasal involvement.

A 33-year-old man suddenly develops severe dyspnea with wheezing. On physical examination, his vital signs are temperature, 37° C; pulse, 95/min; respirations, 35/min; and blood pressure, 130/80 mm Hg. A chest radiograph shows increased lucency in all lung fields. Arterial blood gas analy- sis shows Po2, 65 mm Hg; Pco2, 30 mm Hg; and pH, 7.48. A sputum cytologic specimen shows Curschmann spirals, Char- cot-Leyden crystals, branching septate hyphae, and eosino- phils in a background of abundant mucus. What is the most likely risk factor predisposing him to this illness? A Cytokine gene polymorphisms B Foreign body aspiration C Inhalation of environmental inorganic dusts D Inheritance of a CFTR gene mutation E Reduced circulating α1-antitrypsin levels F Smoking cigarettes for >10 years

A Cytokine gene polymorphisms Asthma, particularly extrinsic (atopic) asthma, is driven by a type I hypersensitivity response and is associated with an excessive TH2 and TH17 cell-mediated immune response. Genetic factors are important in the pathogenesis of atopic asthma and linkage to cytokine genes that map on 5q are strongly associated with development of asthma and other atopic allergies. The Charcot-Leyden crystals represent the breakdown products of eosinophil granules. The Curschmann spirals represent the whorls of sloughed surface epithelium within the abundant mucin. The septated hyphae are Aspergillus organisms colonizing the tracheobronchial tree (allergic bronchopulmonary aspergillosis). Foreign body aspiration may result in inflammation, but without eosinophils. Inorganic dust inhalation leads to restrictive, not obstructive, lung disease. CFTR mutations with cystic fibrosis lead to chronic widespread bronchiectasis. Inflammation with eosinophils is not a significant component of emphysema related to α1-antitrypsin deficiency or to smoking.

A 65-year-old man has had no major medical problems prior to the past year, when he noted increasing malaise along with an 8 kg weight loss. He is a non-smoker. He currently does not have fever, cough, dyspnea, or any respiratory difficulties. On physical examination, he has non-tender supraclavicular lymphadenopathy. The lungs are clear to auscultation. A chest x-ray shows multiple solid nodules ranging from 1 to 3 cm scattered throughout all lung fields. No infiltrates or areas of consolidation are noted. Laboratory studies show Hgb 11.6 g/dL, Hct 34.7%, MCV 83 fL, and WBC count 6280/microliter. Which of the following pathologic processes in his lungs is most likely to account for these findings? A Pulmonary infarctions B Recurrent aspiration C Metastatic carcinoma D Nocardia asteroides infection E Silicosis

C Metastatic carcinoma Multiple persistent masses should suggest metastases, rather than a primary lung tumor. His lack of a cough or fever is against an infectious cause or aspiration.

A clinical study is conducted in which patients who have undergone surgical procedures with intubation, me- chanical ventilation, and general anesthesia are followed to determine the number and type of postoperative complica- tions. The study group is found to have a higher incidence of pulmonary infections in the 2 weeks following their surgical procedure than patients who were not intubated and did not receive general anesthesia. Anesthesia is most likely to pro- duce this effect via which of the following mechanisms? A Decreased ciliary function B Diminished macrophage activity C Hypogammaglobulinemia D Neutropenia E Squamous metaplasia F Tracheal erosions

A Decreased ciliary function The anesthetic gases tend to reduce the ciliary function of the respiratory epithelium that lines the bronchi. The mucociliary apparatus helps clear organisms and particulate matter that are inhaled into the respiratory tree. Macrophage function is not significantly affected by anesthesia. The levels of γ-globulins in serum are not reduced by the effects of anesthesia. The anesthetic gases and induction drugs do not typically result in marrow failure with neutropenia. The exposure to anesthetic gases is not prolonged enough to produce squamous metaplasia, which most typically occurs in response to chronic irritation, as from cigarette smoke. The subglottic tracheal region, where the cuff of the endotracheal tube is located, can become eroded, but this is more likely to occur when intubation is prolonged for weeks.

A 6-year-old child has the sudden onset of dyspnea with wheezing. On physical examination he is afebrile but has absent breath sounds on the right. His temperature is 37°C, pulse 82/minute, respiratory rate 32/minute, and blood pressure 100/60 mm Hg. An arterial blood gas measurement shows pO2 95 mm Hg, pCO2 25 mm Hg, and pH 7.55. Following administration of 100% FiO2 by nasal canula, a repeat measurement shows pO2 95 mm Hg, pCO2 25 mm Hg, and pH 7.55. Which of the following is the most likely diagnosis? A Foreign body aspiration B Bronchial asthma C Paraseptal emphysema D Thromboembolism E Carcinoid tumor

A Foreign body aspiration An inhaled object could obstruct a bronchus completely, with air resorbtion and collapse of lung distal to the point of obstruction. This produces a shunt defect with a V/Q mismatch. Since there is perfusion but no ventilation, even 100% oxygen will not make a difference. His hyperventilation has acutely produced an uncompensated respiratory alkalosis.

A 55-year-old man with a 50 pack year history of smoking cigarettes has recently experienced an episode of hemoptysis along with his usual cough. On physical examination he has no abnormal findings. A sputum for cytology on microscopic examination shows atypical cells with hyperchromatic nuclei and orange-pink cytoplasm. Labortory studies show a serum calcium of 11.3 mg/dL, with phosphorus 2.1 mg/dL. Which of the following chest radiographic findings is this man most likely to have? A Large hilar mass B Pneumonia-like consolidation C Peripheral nodule D Carinal compression E Left pleural thickening

A Large hilar mass Both squamous cell and small cell carcinomas, most common in smokers, tend to be central in location. Squamous cell carcinomas of the lung are associated with hypercalcemia, though overall the small cell carcinomas are best known for paraneoplastic syndromes B - This is more typical of adenocarcinoma-in-situ. C - Such a peripheral mass would more likely be a granuloma, an adenocarcinoma or a hamartoma. E - The findings do not suggest a mesothelioma.

A 46-year-old woman has a routine health maintenance examination. On physical examination, there are no remark- able findings. Her body mass index is 22. She does not smoke. A tuberculin skin test is positive. A chest radiograph shows a solitary, 3-cm left upper lobe mass without calcifications. The mass is removed at thoracotomy by wedge resection. The microscopic appearance of this lesion is shown in the figure. Which of the following is the most likely diagnosis? A Mycobacterium tuberculosis infection B Necrotizing granulomatous vasculitis C Poorly differentiated adenocarcinoma D Staphylococcus aureus abscess E Thromboembolism with infarction

A Mycobacterium tuberculosis infection The figure shows pink, amorphous tissue at the lower left, representing caseous necrosis. The rim of the granuloma has epithelioid cells and Langhans giant cells. Caseating granulomatous inflammation is most typical of Mycobacterium tuberculosis infection. Calcifications would have helped to identify this mass as an old granuloma, not likely to be a neoplasm. Necrotizing vasculitis is unlikely to produce a single nodule, and there can be hemoptysis. A carcinoma may have central necrosis, not caseation, and there would be atypical, pleomorphic cells forming the mass. A pulmonary infarct should have extensive hemorrhage. A lung abscess would have an area of liquefactive necrosis filled with tissue debris and neutrophils.

A 40-year-old woman has had a high fever for a week, accompanied by a cough productive of yellowish sputum. On physical examination her temperature is 38.2°C. There are diffuse rales in all lung fields. Her chest radiograph reveals patchy infiltrates in all lung fields, and there is a 4 cm rounded area of consolidation in the left upper lobe that has an air-fluid level. Examination of her sputum reveals numerous neutrophils. Which of the following infectious agents is most likely causing her pulmonary disease? A Staphylococcus aureus B Aspergillus niger C Mycobacterium tuberculosis D Mycoplasma pneumoniae E Adenovirus

A Staphylococcus aureus Pulmonary abscesses typically have an air-fluid level from liquefactive necrosis caused by the collection of neutrophils. More virulent bacterial organisms such as Staphylococcus aureus are likely to cause such a complication of a bronchopneumonia. B - Fungus balls are usually firm masses that do not have central necrosis and/or cavitation. They may have adjacent hemorrhage as fungal hyphae invade into the surrounding vasculature. C - Although cavitation is a feature of secondary tuberculosis, air-fluid levels within the cavities are not frequent due to the caseous nature of the necrosis. D - Infection with Mycoplasma pneumoniae typically produces patchy infiltrates, but abscess formation is not likely to occur, and the process is more interstitial with mononuclear cells. E - Adenovirus produces an interstitial pneumonitis. However, viral pneumonias may be complicated by secondary bacterial pneumonias.

A 26-year-old woman from East Asia developed a fever with chills over the past 4 days. Yesterday, she had increasing shortness of breath and a nonproductive cough, headache, and myalgias. On physical examination, her temperature is now 38.6° C. A chest radiograph shows right lower lobe infiltrates. Laboratory studies show hemoglobin, 13.4 g/dL; hematocrit, 40.2%; platelet count, 78,400/mm3; and WBC count, 3810/mm3 with 77% segmented neutrophils, 2% bands, 5% lymphocytes, and 16% monocytes. Over the next 2 days, she has increasing respiratory distress requiring intubation and mechanical ven- tilation. A repeat chest radiograph shows worsening bilateral infiltrates. Infection with which of the following is most likely to have caused this patient's illness? A Coronavirus B Cytomegalovirus C Ebola virus D Herpes simplex virus E Respiratory syncytial virus

A. Coronavirus Severe acute respiratory syndrome is caused by a strain of coronavirus that is much more virulent than the coronaviruses known to be associated with the common cold. Cytomegalovirus is seen in immunocompromised patients and often involves multiple organs. Ebola virus is virulent and does not cause specific respiratory findings. Herpes simplex virus is a very rare cause of pneumonia, even in immunocompromised patients. Respiratory syncytial virus causes acute respiratory illness in young children.

A 28-year-old man presents with 6 days of fever and shortness of breath. His temperature is 38.7°C (103°F), respirations are 30 per minute, and blood pressure is 120/80mmHg. A chest X-ray reveals diffuse interstitial and alveolar infiltrates. Spu- tum cultures are negative, and the patient does not respond to standard antibiotic therapy. A transbronchial lung biopsy is shown in the image. Which of the following is the appropriate diagnosis? (A) Eosinophilic pneumonia (B) Lipid pneumonia (C) Pneumococcal pneumonia (D) Pneumocystis pneumonia (E) Usual interstitial pneumonia

A. Eosinophilic pneumonia Principally an allergic disorder. It refers to the accumulation of eosinophils in alveolar spaces and is classified as either idiopathic or secondary to an underlying illness. In acute eosinophilic pneumonia, the alveolar spaces are filled with an inflammatory exudate composed of eosinophils and macrophages. The alveolar septa are thickened by the presence of numerous eosinophils and hyaline membranes are present. Patients respond dramatically to corticosteroids, and, in contrast to chronic eosinophilic pneumonia, acute eosinophilic pneumonia does not recur. Excess eosinophils are not encountered in the other choices. Diagnosis: Eosinophilic pneumonia

A 10-year-old male presents with a prolonged history of cough and hemoptysis. Laboratory testing is negative for anti-GBM and ANCA antibodies. Histologic features of this disease include: A. Numerous hemosiderin laden macrophages B. Alveolar septal necrosis and hemorrhage C. Granulomatous vasculitis D. Hyaline membranes E. Necrotizing granulomas

A. Numerous hemosiderin laden macrophages (idiopathic pulmonary hemosiderosis)

A 29-year-old man who has had no major illnesses expe- riences acute onset of hemoptysis. On physical examination, he has a temperature of 37° C, pulse of 83/min, respirations of 28/min, and blood pressure of 150/95 mm Hg. A chest radio- graph shows bilateral fluffy infiltrates. A transbronchial lung biopsy on microscopic examination shows focal necrosis of al- veolar walls associated with prominent intra-alveolar hemorrhage. Two days later, he has oliguria. The serum creatinine level is 2.9 mg/dL, and urea nitrogen is 31 mg/dL. Which of the following antibodies is most likely involved in the patho- genesis of his condition? A Anti-DNA topoisomerase I antibody B Anti-glomerular basement membrane antibody C Antimitochondrial antibody D Anti-neutrophil cytoplasmic antibody E Antinuclear antibody

B - Anti-glomerular basement membrane antibody Goodpasture syndrome leads to renal and pulmonary lesions produced by an antibody directed against an antigen common to the basement membrane in glomerulus and alveolus. This leads to a type II hypersensitivity reaction. The anti-DNA topoisomerase I antibody is a marker for scleroderma. Antimitochondrial antibody is associated with primary biliary cirrhosis. C-ANCA and P-ANCA are best known as markers for various forms of systemic vasculitis. ANA is used as a general screening test for various autoimmune conditions, typically collagen vascular diseases such as systemic lupus erythematosus.

A 61-year-old man has had a cough without production of much sputum for the past week. On physical examination he is afebrile. There are decreased breath sounds at the right lung base. A chest x-ray reveals an area of consolidation in the right lower lobe. He is given antibiotic therapy, but a month later the radiographic picture has not changed, and his cough continues. A bronchoalveolar lavage is performed and yields atypical cells along with scattered alveolar macrophages. Which of the following is the most likely diagnosis? A Mycoplasma pneumonia B Adenoccarcinoma-in-situ C Sarcoidosis D Pulmonary infarction E Silica inhalation

B Adenocarcinoma-in-situ Adenocarcinoma-in-situ can spread in a pneumonia-like pattern. The lack of a change over time and the absence of a response to antibiotics should suggest an underlying non-infectious process. Adenocarcinomas may occur in non-smokers. A - An atypical pneumonia with Mycoplasma is unlikely to be confined to just the right lower lobe, and it would not yield atypical cells. It could respond to antibiotic therapy, such as erythromycin. C - Sarcoid is likely to produce a granulomatous pattern with reticulonodular infiltrates, and the hilar lymph nodes are usually enlarged as well. D - An infarct is unlikely to yield atypical cells. A pulmonary infarct is most likely to follow pulmonary thromboembolism in a person with pre-existing pulmonary or cardiac disease. E - Silicosis produces interstitial lung disease with silicotic nodules.

A 70-year-old woman has had episodes of dyspnea with wheezing and coughing, accompanied by urticaria for the past 3 years. She has had bouts of rhinitis. She has a 10-year his- tory of osteoarthritis. On physical examination she has nasal polyps. Use of which of the following medications is the most likely risk factor for her respiratory disease? A Acetaminophen B Aspirin C Gabapentin D Morphine E Prednisone

B Aspirin Drug-induced asthma is most likely to occur in older patients who develop increased sensitivity to a drug. Aspirin (acetylsalicylic acid) blocks the cyclooxygenase pathway of arachidonic acid metabolism but not the lipoxygenase pathway that potentiates bronchoconstriction. NSAIDs may have the same effect as aspirin. Angiotensin-converting enzyme (ACE) inhibitors may also induce asthmalike episodes. Acetaminophen is an analgesic that can be substituted for aspirin and is unlikely to provoke asthmatic attacks. Gabapentin and morphine act centrally as analgesics. Prednisone is an anti-inflammatory agent that is used to treat immune-mediated diseases.

A 40-year-old man has had an increasing cough with he- moptysis for 2 weeks. He has never smoked and is in very good health. On physical examination, his temperature is 38.2° C. A chest radiograph shows an area of consolidation in the right upper lobe. His condition improves with antibiotic therapy; however, the cough and hemoptysis persist for 2 more weeks. Chest CT scan now shows right upper lung atelectasis. Bron- choscopic examination shows a tan, circumscribed obstructive mass filling a right upper lobe bronchus. Which of the follow- ing neoplasms is most likely to produce these findings? A Adenocarcinoma B Carcinoid tumor C Hamartoma D Kaposi sarcoma E Large cell carcinoma

B Carcinoid tumor Most pulmonary carcinoids are central obstructing masses involving a large to medium-sized bronchus. These neuroendocrine tumors have unpredictable behavior, but many are localized, resectable, and follow a benign course. They typically manifest with hemoptysis and the consequences of bronchial obstruction. In this case, the pneumonia in the right upper lobe probably resulted from obstruction to drainage caused by the tumor. Obstruction may lead to peripheral resorption atelectasis. Adenocarcinomas are common lung tumors, but are typically peripheral. A hamartoma is an uncommon but benign pulmonary lesion that also is located peripherally. Capos sarcoma can involve the lung in some patients with AIDS, and the tumor often has a bronchovascular distribution; obstruction is not common, but bleeding on biopsy is. Large-cell carcinomas are typically large, bulky, peripheral masses.

A 46-year-old man from northern Mexico has had fever, nonproductive cough, and weight loss for 2 months. On exam- ination his temperature is 37.5 ° C. A chest radiograph shows a miliary pattern of small nodules in all lung fields. Bronchoalveolar lavage is performed and microscopic examination of the fluid shows organisms averaging 50 microns in diameter with thick walls and filled with endospores. Which of the following infections is he most likely to have? A Blastomycosis B Coccidioidomycosis C Histoplasmosis D Mycobacteriosis E Nocardiosis F Paracoccidioidomycosis

B Coccidioidomycosis Coccidioidomycosis is endemic to semiarid regions of the Americas. The species Coccidioides immitis is found in the southwestern United States, and the nearly identical Coccidioides posadasii is found from Mexico to South America. Dimorphic fungal diseases may produce granulomatous infection resembling tuberculosis. The miliary pattern described in this patient occurs with a poor immuneresponse. Blastomyces dermatitidis organisms are smaller than 10 microns and exhibit broad-based budding. Histoplasma capsulatum organisms are 2 to 4 microns and are often found within phagocytic cells. Mycobacteria are acid fast rod-shaped organisms only a micron across. Nocardia organisms are long, filamentous bacteria that may be weakly acid-fast. Paracoccidioides brasiliensis organisms can be 20 to 30 microns and exhibit multiple buds in a "captain's wheel" configuration.

A 68-year-old man has had increasing dyspnea with cough productive of frothy sputum for the past 5 months. On physical examination, he is afebrile, and his blood pressure is 165/100 mm Hg. There is dullness to percussion at lung bases. He has pitting edema of the ankles. A chest radiograph shows blunting of costophrenic recesses bilaterally and car- diomegaly with prominent right and left heart borders. A right thoracentesis is performed, and 300 mL of straw-colored fluid is removed. Laboratory studies on this fluid show total protein of 2.2 g/dL (serum is 6.5 g/dL), glucose of 45 mg/dL (serum is 75 mg/dL), lactate dehydrogenase of 200 U/L (se- rum is 420 U/L), pH 7.2, and cell count of 100/mm3 mononu- clear leukocytes, and no RBCs. What condition does he most likely have? A Cavitary tuberculosis B Congestive heart failure C Malignant mesothelioma D Non-Hodgkin lymphoma E Pneumococcal pneumonia F Small cell carcinoma

B Congestive heart failure The pleural fluid findings with low protein, low LDH, and low cell count are consistent with a transudate with hydrothorax. His hypertension has likely led to left ventricular failure with pulmonary edema, and longstanding left ventricular failure can lead to right ventricular failure with body cavity effusions and peripheral edema. Granulomatous diseases, including tuberculosis, along with carcinomas that are primary or metastatic to pleura, tend to produce cellular effusions with numerous RBCs. Mesothelioma is likely to shed numerous cells into the pleural cavity, although in most cases there is dense tumor that obliterates the pleural cavity. Lymphomas may block lymphatics, including the thoracic duct, to produce a milky chylothorax with lipid and leukocytes. Bacterial pneumonias spreading to the pleura are likely to produce an exudative effusion with numerous leukocytes, predominantly neutrophilic.

An epidemiologic study shows that a highly pathogenic strain of influenza A virus with the antigenic type H5N1 that normally causes disease in birds has been increasingly found to cause influenza in humans. Unlike other strains of influen- za A virus, this H5N1 virus is associated with a 60% mortal- ity rate. The enhanced pathogenicity of this avian flu virus is primarily due to mutation in its genome that enables it to do which of the following? A Elicit a weak cytotoxic T-cell response B Enter many types of host cells C Escape inactivation by macrophages D Infect CD4+ helper T cells E Spread from humans to humans

B Enter many types of host cells The antigenic drift of influenza viruses, by mutation with minor alteration of either their hemagglutinin (H) or neuraminidase (N) genes, allows them to escape host antibodies. A shift occurs with major change in H or N or both, as happened with H1N1 in this case. Cleavage of influenza viral hemagglutinin by host proteases is essential for the virus to enter cells. The less virulent influenza viruses are cleaved by proteases that are mainly localized to the lung and hence the disease is limited to the lungs. H5N1 virus has much broader tissue tropism because its hemagglutinin can be cleaved by proteases present in many tissues. Host responses to flu virus, such as a cytotoxic T-cell response or macrophage engulfment, are not the major determinant of pathogenicity. Selective infection of CD4+ T cells is a propensity of HIV. Currently avian flu cannot be spread from human to human—but should that happen there would be an avian flu pandemic.

A 51-year-old man complains of a slight cough he has had for a week. He is a non-smoker. On auscultation of the chest his lung fields are clear. A chest radiograph shows a subpleural 'coin lesion' 2 cm in diameter in the right upper lobe. Which of the following is the most likely diagnosis for this lesion? A Small cell carcinoma B Granuloma C Bronchiectasis D Exogenous lipid pneumonia E Silicosis

B Granuloma The differential diagnosis of a solitary peripheral coin lesion most often includes lung cancer (adenocarcinoma most likely), granuloma, or hamartoma. Many people have had a remote, subclinical tubercular or fungal infection leaving only a solitary granuloma. In this case the cough suggests possible active infection, but it may just be an incidental, unrelated finding. A - An 'oat cell' carcinoma tends to spread very quickly and not remain localized. Moreover, it virtually always appears in persons with a history of smoking.

A 43-year-old woman has never smoked and works as a file clerk at a university that designates all work areas as non- smoking. A routine chest radiograph shows a 3-cm, sharply demarcated mass in the left upper lobe of the lung. Fine- needle aspiration of the mass is attempted, but the patholo- gist performing the procedure remarks, "This is like trying to biopsy a ping-pong ball." No tissue is obtained. Thoracotomy with wedge resection is performed. On sectioning, the mass has a firm, glistening, bluish white cut surface. A culture of the mass yields no growth. This mass most likely represents which of the following? A Adenocarcinoma B Hamartoma C Large cell carcinoma D Mesothelioma E Non-Hodgkin lymphoma F Squamous cell carcinoma

B Hamartoma Hamartomas are uncommon but benign peripheral lesions of the lung. They are composed of benign-appearing epithelial cells and connective tissue, typically with a large component of cartilage. They are included in the differential diagnosis of a "coin lesion" that also includes carcinoma and granuloma. Adenocarcinoma is the most common primary lung malignancy in nonsmokers, and it can manifest as a coin lesion, but it is composed of gland-forming, malignant cells without cartilage. It tends to be peripheral, making surgical resection an option in many cases. Large cell carcinomas also are more likely to be peripheral, but they tend to be larger masses, with poorly differentiated cells. Malignant mesothelioma is a rare neoplasm, even in individuals who have been exposed to asbestos, and it arises on the pleura. Primary non-Hodgkin lymphomas of the lung are uncommon, but may involve hilar or mediastinal lymph nodes. Some squamous cell carcinomas can be peripheral, but they are most likely to occur in individuals who smoke.

During a cardiac arrest, a 58-year-old man, a non-smoker, receives cardiopulmonary resuscitative measures and is brought to the hospital, where he is intubated. During the intubation procedure he suffers aspiration of gastric contents. Over the next 10 days he develops a non-productive cough along with a fever to 37.9°C. A chest radiograph reveals a 4 cm diameter mass with an air-fluid level in the right lung. A sputum gram stain reveals mixed flora. Which of the following conditions is he most likely to have? A Squamous cell carcinoma B Lung abscess C Chronic bronchitis D Bronchiectasis E Bronchopulmonary sequestration

B Lung abscess Lung abscesses can result from aspiration of oropharyngeal or nasopharyageal contents, where bacterial organisms as part of normal flora can be transported to the lungs. The straighter bronchus to the right lung is more likely to conduct aspirated material. With septicemia, multiple abscesses are more likely to be present. The infection is often polymicrobial and difficult to treat. A - There is no propensity for a lung carcinoma, either primary or metastatic, to be in one lung or another. Lung cancers may have central cavitation if large, but usually not an air-fluid level. Large neoplasms can cause obstruction with pneumonia distal to the neoplasm.

An 11-year-old girl has experienced dyspnea for the past 2 weeks. Her temperature is 37.3°C, pulse 85/minute, respiratory rate 30/minute, and blood pressure 110/60 mm Hg. On physical examination, her lung fields are clear to auscultation. Her heart rate is regular and no murmurs or gallops are heard. A chest radiograph shows prominent hilar lymphadenopathy along with a 1 cm peripheral right middle lobe nodule. No infiltrates are present. A sputum gram stain shows normal flora and routine bacterial culture reveals no pathogens. Which of the following conditions is she most likely to have? A Hypersensitivity pneumonitis B Mycobacterium tuberculosis infection C Bronchial carcinoid tumor D Infective endocarditis E Goodpasture syndrome

B Mycobacterium tuberculosis infection This is a description of the typical 'Ghon complex' of an initial, or primary, TB infection. The peripheral granuloma does not produce symptoms, but the enlarged lymph nodes may impinge upon airways. Most persons with a primary infection, however, are asymptomatic and the infection goes unnoticed. E - Goodpasture syndrome is rare at this age, and the pattern is that of diffuse pulmonary hemorrhage with hemoptysis.

A 38-year-old previously healthy woman has had a worsening non-productive cough for the past 4 days. On physical examination her temperature is 38.3°C. A chest radiograph shows patchy infiltrates and diffuse interstitial markings. Laboratory studies show a sputum gram stain with mixed flora. Her Hgb is 12.9 g/dL, platelet count 229,450/microliter, and WBC count 5815/microliter. Her cold agglutinin titer is elevated. Following a course of erythromycin therapy, she improves, with no complications. Which of the following infectious agents is the most likely cause for her pulmonary disease? A Nocardia asteroides B Mycoplasma pneumoniae C Mycobacterium kansasii D Respiratory syncytial virus E Chlamydia psittici F Adenovirus G Klebsiella pneumoniae

B Mycoplasma pneumoniae Mycoplasma infection predominantly affects the interstitium, and is not a widespread alveolar filling process. It is a cause for a 'primary atypical pneumonia' which is difficult to diagnose because this organism is not cultured by routine methods for bacterial organisms. The cold agglutinin titer is elevated in about half of cases and is a characteristic finding. Many cases respond to antibiotic therapy with erythromycin. A - Nocardia braziliensisInfections occur in immunocompromised hosts and produce a chronic abscessing process. D - Psitticosis is rare and can lead to a severe pneumonia known as ornithosis. It does not respond to erythromycin therapy. F - Adenovirus can produce interstitial infiltrates, but does not respond to erythromycin. G - Klebsiella is a gram negative bacterium that most often produces a bronchopneumonia with productive cough and neutrophilia.

A 3-day-old girl shows signs of cyanosis and respiratory distress. Her temperature is 38.7°C (103°F), pulse rate is 140 per minute, respirations are 60 per minute, and blood pressure is 90/58mmHg. Laboratory studies indicate that the baby is positive for HIV. The infant does not respond to conventional antibiotic therapy and expires. Histo- logic examination of the lungs at autopsy is shown in the image. The alveolar cells are very large and display single basophilic nuclear inclusions, with a peripheral halo and multiple cytoplasmic basophilic inclusions. Which of the following is the most likely etiologic agent in this child's pulmonary infection? (A) Adenovirus (B) Cytomegalovirus (CMV) (C) Herpesvirus (D) Pneumocystis carinii (E) Rubellavirus

B. CMV CMV produces a characteristic interstitial pneumonia. Initially described in infants, it is now well recognized in immunocompromised persons. The virus may be transmitted from mother to child transplacentally (a feature of TORCH syndrome). Although infected children are usually asymptomatic, in symptomatic infants and children, central nervous symptoms predominate. As implied by its name, CMV causes marked enlargement of infected cells, which contain typical intranuclear and often cytoplasmic inclusions. The other TORCH agents may cause pneumonia, but only CMV exhibits this cellular morphology. Diagnosis: Cytomegalovirus, viral pneumonia

A 23-year-old female presents to the emergency room with severe pulmonary hemorrhage. Laboratory studies are positive for anti-GBM antibodies. The patient's history is likely positive for: A. Alcohol B. Smoking C. Weight loss D. Bacterial pneumonia E. Positive family history

B. Smoking

A 44-year-old woman, a non-smoker, has had a fever and cough for the past 4 days. She does not have hemoptysis. She has not experienced weight loss, malaise, nausea, or vomiting. On physical examination her temperature is 37.6°C. There are decreased breath sounds over the right upper lung. A chest radiograph reveals a 6 cm area of infiltrates in the right upper lobe. She is given a course of antibiotic therapy, but her cough persists. A month later her chest x-ray now reveals a 3 cm peripheral mass in the right upper lobe. Which of the following neoplasms is most likely to be present in this woman? A Squamous cell carcinoma B Small cell anaplastic carcinoma C Adenocarcinoma D Mesothelioma E Carcinoid tumor

C Adenocarcinoma Peripheral lung cancers (adenocarcinoma and large cell carcinoma) show less of an association with smoking than central lung cancers (small cell and squamous cell carcinoma). The focal obstruction from the mass predisposed to infection. A - Squamous cell carcinomas are often associated with smoking. They are often bulky central masses. B - Oat cell carcinomas have a close association with smoking. Such cancers are not common in nonsmokers. They often metastasize early, so that the primary site (usually in a central location) is small when discovered. D - Asbestos exposure is a likely antecedent to pleural mesothelioma, which produces a bulky pleural mass. E - Carcinoid tumors are neuroendocrine neoplasms on the opposite end of the spectrum from oat cell carcinomas. Carcinoids tend to form masses within the larger bronchi.

A 72-year-old woman has had difficulty with vision in her right eye for 3 months. She also has pain in the right upper chest. The findings on physical examination include unilateral enophthalmos, miosis, anhidrosis, and ptosis on the right side of her face. A chest radiograph shows right upper lobe opacifi- cation and bony destruction of the right first rib. Which of the following conditions is most likely to be present in her? A Bronchopneumonia B Bronchiectasis C Bronchogenic carcinoma D Sarcoidosis E Tuberculosis

C Bronchogenic carcinoma Horner syndrome is a result of sympathetic autonomic nerve involvement by invasive pulmonary carcinoma. Such a neoplasm in this location with these associated findings is called a Pancoast tumor. Infectious processes such as pneumonia are unlikely to impinge on structures outside the lung. Bronchiectasis destroys bronchi within the lung. Sarcoidosis can result in marked hilar adenopathy with a mass effect, but involvement of the peripheral nervous system is unlikely. Likewise, tuberculosis is a granulomatous disease that can lead to hilar adenopathy, although usually without destruction of extrapulmonary tissues.

A 25-year-old man receives a hematopoietic stem cell transplant for treatment of acute myelogenous leukemia. He develops increasing dyspnea 3 weeks later, along with fever and cough. On physical examination his temperature is 37.8°C. A chest radiograph shows irregular interstitial infiltrates. A bronchoalveolar lavage is performed an on cytologic examination shows cells that are enlarged and have prominent intranuclear inclusions. He is most likely to have an infection with which of the following organisms? A Toxoplasma gondii B Candida albicans C Cytomegalovirus D Pneumocystis jiroveci E Mycobacterium avium complex F Influenza B virus G Respiratory syncytial virus

C Cytomegalovirus CMV produces a cytopathic effect with enlarged cells having prominent intranuclear inclusions. CMV infection occurs most often with immune compromise, as in this case folowing transplantation before engraftment. D - PCP yields foamy material that appears acellular with H&E and Pap stains. E - Mycobacteria produce a granulomatous reaction. There may be giant cells of the Langhans variety with MTB, and just large macrophages will with acid fast bacilli in MAC, but a BAL will probably not have much that is diagnostic without an AFB stain or culture. F - Influenza virus infections are not characteristic for immunocompromised patients. No characteristic inclusions can be seen in specimens. G - Respiratory syncytial virus infections are most common in young children and infants. They are not characteristic for immunocompromised patients. Inclusions are in the cytoplasm with this RNA virus.

A 47-year-old woman known to be HIV-positive has had a decreasing CD4+ lymphocyte count for the past year despite antiretroviral therapy. She has developed a fever with nonproductive cough over the past week. On auscultation of her chest fine crackles are present in both lungs. A chest radiograph shows infiltrates in both lungs. A transbronchial biopsy is obtained and the microscopic appearance is shown in the figure. Which of the following organisms is most likely infecting this woman? A Candida albicans B Cryptococcus neoformans C Cytomegalovirus D Klebsiella pneumoniae E Pneumocystis jiroveci

C Cytomegalovirus Within the airspace are multiple large cells with prominent purple intranuclear inclusions, typical for CMV. Although CMV antibodies can be found in most immunocompetent persons, the infection is subclinical. However, in immunocompromised persons, CMV infection can be a severe and systemic infection, including marked interstitial pneumonitis. The other listed organisms are much smaller. Candida is a rare cause for pneumonia, and this yeast appears as budding cells and pseudohyphae. Cryptococcal organisms exhibit narrow-based budding and thick mucoid capsules. Klebsiella pneumoniae organisms are encapsulated, but these bacteria are too small to be seen with H&E staining. Pneumocystis organisms are best seen with Gomori methenamine silver stain and appear as round to cup-shaped cysts.

A 43-year-old woman has had malaise and an 8-kg weight loss over the past 3 years. She has had fever and a nonproductive cough with increasing dyspnea for the past 3 days. On physical examination, her temperature is 37.8° C. There is dullness to percussion over the lungs and diffuse crackles on auscultation. A chest radiograph shows extensive bilateral infiltrates. Bron- choalveolar lavage is done, and the fluid is stained with Gomori methenamine silver, with high-power microscopic appearance shown in the figure. Which of the following underlying condi- tions is most likely present in this woman? A Centrilobular emphysema B Diabetes mellitus C HIV infection D Sarcoidosis E Severe combined immunodeficiency F Systemic lupus erythematosus

C HIV infection Although Pneumocystis jiroveci pneumonia can be seen with various acquired and congenital immunodeficient states (mainly those affecting cell-mediated immunity), it is most often associated with AIDS and is diagnostic of AIDS in HIV-infected individuals. Persons with diabetes mellitus and pulmonary emphysema are most prone to contract bacterial infections. Patients with autoimmune diseases may have cytopenias that predispose to infection, and if they are treated with immunosuppressive drugs, various infections are possible. Likewise, patients with sarcoidosis treated with corticosteroid therapy may have opportunistic infections. A patient with severe combined immunodeficiency is susceptible to P. jiroveci pneumonia, but it is unlikely that without treatment such a patient would have survived to age 40 years.

A 66-year-old woman has had a worsening non-productive cough with malaise for the past week. Her temperature increases to 37.4°C. A chest radiograph reveals diffuse bilateral pulmonary interstitial infiltrates in all lung fields. A sputum gram stain reveals normal flora and few neutrophils. She recovers over the next two weeks without sequelae. Infection with which of the following organisms most likely caused her illness? A Mycobacterium tuberculosis B Streptococcus pneumoniae C Influenza A virus D Cryptococcus neoformans E Mycobacterium avium-complex

C Influenza A virus The typical appearance of a viral lung infection is interstitial inflammation. The inflammatory response is primarily T lymphocytes. A - TB most often produces a granulomatous pattern of inflammation with reticulonodular densities, particularly in the upper lobes. Clinically apparent disease tends to be chronic.. B - Pneumococcus leads to alveolar filling with neutrophils, leading to a productive cough with purulent sputum. D - Cryptococcus neoformans is a fungus that leads to granulomatous disease. Though it may occur in immuncompetent hosts, it is best known as an opportunistic infection. E - MAC may produce poorly-formed granulomas and is more common in immunocompromised hosts.

A previously healthy, 20-year-old woman has had a low-grade fever for the past 2 weeks. On physical examination, her temperature is 37.7° C; there are no other remarkable findings. The gross appearance of the lung shown in the figure is representative of her disease. Which of the following laboratory studies is most likely to report a positive result? A Anticentromere antibody B HIV serologic test C Interferon-γ release assay D Rapid plasma reagin E Rheumatoid factor F Sweat chloride

C Interferon-γ release assay These findings represent the so-called Ghon (or primary) complex, consisting of a small sub pleural granuloma with extensive hilar nodal caseating granulomas. The Ghon complex is a feature of primary tuberculosis, which is most often a subclinical disease of younger individuals. The interferon-γ release assay is positive when T lymphocytes are activated against Mycobacterium tuberculosis antigens, and is negative in individuals who received BCG vaccination. Individuals who are immunocompromised, such as HIV-infected patients, do not mount a good granulomatous response and have more extensive poorly formed granulomas, dissemination of tuberculosis, or both. Anticentromere antibody is characteristic of limited scleroderma, which does not have significant pulmonary involvement, in contrast to diffuse scleroderma. The rapid plasma reagin test is used to diagnose syphilis, which does not have significant pulmonary disease. Rheumatoid nodules may be seenin rheumatoid arthritis; these can be subpleural, but patients typically have arthritis. Patients with cystic fibrosis and an elevated sweat chloride level (more often elevated in children than in adults) develop widespread bronchiectasis along with infection by bacterial agents, particularly Pseudomonas aeruginosa and Burkholderia cepacia.

A 3-year-old boy has had a cough, headache, and slight fever for 5 days. He is now having increasing respiratory dif- ficulty. On physical examination, his temperature is 37.8° C, pulse is 81/min, respirations are 25/min, and blood pressure is 90/55 mm Hg. On auscultation, there are inspiratory crack- les, but no dullness to percussion or tympany. Respiratory syncytial virus is isolated from a sputum sample. Which of the following chest radiographic patterns is most likely to be present? A Hilar lymphadenopathy B Hyperinflation C Interstitial infiltrates D Lobar consolidation E Pleural effusions F Upper lobe cavitation

C Interstitial infiltrates Respiratory syncytial virus (RSV) pneumonia is most common in children, and it can occur in epidemics. Viral, chlamydial, and mycoplasmal pneumonias are most often interstitial, without neutrophilic alveolar exudates. The diagnosis is often presumptive because viral culture is technically difficult and expensive, such as a PCR assay. Hyperinflation can accompany bronchoconstriction in asthma. Marked lymphadenopathy is more characteristic of chronic processes, such as granulomatous diseases or metastases. Lobar consolidation is more typical of a bacterial process, such as can be seen in Streptococcus pneumoniae infection. Pleural effusions can be seen in pulmonary inflammatory processes, but they are most pronounced in heart failure. Cavitation is most likely to complicate secondary tuberculosis in adults.

A 71-year-old woman has smoked a pack of cigarettes per day for 50 years. She has had increasing dyspnea for 12 years. Over the past 3 days she has become febrile, with a productive cough, and severe dyspnea. Auscultation of her chest reveals rales and expiratory wheezes. Laboratory studies show peripheral blood neutrophilia. Which of the following organisms is most likely to be cultured from her sputum? A Histoplasma capsulatum B Influenza A C Moraxella catarrhalis D Mycobacterium kansasii E Pneumocystis jiroveci

C Moraxella catarrhalis The short time course and acute inflammatory response are consistent with bacterial pneumonitis. Moraxella is the only bacterial organism listed and is in the differential diagnosis of both upper and lower respiratory infections, particularly in persons with chronic obstructive pulmonary disease (COPD), such as this woman. Moraxella is an aerobic gram-negative diplococcus that is oxidase positive. Histoplasmosis is typically a granulomatous disease that develops over weeks. Influenza A viral infection is interstitial, without neutrophilia. M. kansasii produces pulmonary granulomatous disease similar to tuberculosis. Pneumocystis pneumonia is most often seen in immunocompromised persons

A 41-year-old man with a 6 kg weight loss over the past 3 months now has had worsening fever, non-productive cough, and dyspnea for the past 3 days. His temperature is 38.2°C and there are diffuse rales in both lungs on auscultation. A chest radiograph shows patchy infiltrates in both lungs. Laboratory studies show WBC count 3250/microliter with 81 segs, 3 bands, 5 lymphs, and 11 monos. His CD4 lymphocyte count is 79/microliter. Cryptosporidium parvum organisms are found in a stool specimen. A bronchoalveolar lavage is performed, yielding fluid that microscopically demonstrates pink, foamy exudate with little inflammation. Which of the following additional findings on microscopic examination is he most likely to have in the BAL specimen? A Acid fast bacilli B Branching septate hyphae C Multiple cysts with GMS stain D Hemosiderin-laden macrophages E Short gram positive rods

C Multiple cysts with GMS stain He is most likely to have Pneumocystis jiroveci pneumonia in association with the acquired immunodeficiency syndrome (AIDS). Note his lymphopenia from the very low CD4 count. PCP infection produces an exudate composed of the Pneumocystis cysts and trophozoites with little accompanying inflammation. The clinical findings in this case are typical as well. A - Mycobacterial infection is unlikely in this setting, for there is still typically caseous necrosis and even poorly formed granulomas in an immunocompromised person with M. tuberculosis. M. avium-complex is less likely to produce florid pulmonary disease. B - Aspergillus often produces a fungus ball, or a defined mass, rather than infiltrates. It does not produce an acellular exudate. D - Pulmonary hemorrhage in the setting of AIDS is not common. The clinical features suggest an acute infection. E - Short gram positive rods suggest listeriosis, a disease that can sometimes be seen in immunocompromised persons, but it is not common, and it produces focal abscesses or small granulomas, not foamy exudate.

A 20-year-old man has had a mild fever with nonpro- ductive cough, headache, and myalgias for the past week. On physical examination he has a temperature of 37.9° C and ery- thema of the pharynx. Diffuse crackles are heard on ausculta- tion of the lungs. A chest radiograph shows bilateral extensive patchy infiltrates. A sputum Gram stain shows normal flora. Cold agglutinin titer is elevated. He receives a course of eryth- romycin therapy, and his condition improves. Infection with which of the following organisms is most likely to produce these findings? A Legionella pneumophila B Mycobacterium fortuitum C Mycoplasma pneumoniae D Nocardia asteroides E Respiratory syncytial virus

C Mycoplasma pneumonia This primary atypical pneumonia is caused by Mycoplasma pneumoniae, a cell wall-deficient organism that is difficult to culture. Often, a diagnosis is made empirically. The findings are similar to those of other viral infections, and serologic testing shows the specific organism. Legionella can produce an extensive pneumonia with neutrophilic alveolar exudates, and the organisms are difficult to show—they may be revealed by Dieterle silver stain. Mycobacterium fortuitum is a rare infection that is most likely to be seen in very ill or immunocompromised individuals. Nocardiosis produces chronic abscessing inflammation; it is seen mostly in immunosuppressed individuals. Respiratory syncytial virus is typically an infection of early childhood.

A 58-year-old man has developed a non-productive cough worsening over the past 2 months. Last week he noted the appearance of blood-streaked sputum. On physical examination there are some expiratory wheezes auscultated over the left lung. A chest radiograph reveals a 5 cm mass near the left lung hilum. A sputum cytology reveals the presence of small clusters of very hyperchromatic, pleomorphic cells with scant cytoplasm. Which of the following is the most likely predisposing factor to development of his pulmonary disease? A Silicosis B Radon gas exposure C Smoking D Asbestosis E Chronic bronchitis

C Smoking Smoking remains the most frequent cause of lung cancer. Lung cancer does, however, occur in nonsmokers. A small cell anaplastic carcinoma, as in this patient, is virtually always seen in smokers. Small cell cancers arise in the central portion of the lung but are aggressive neoplasms that spread quickly.

A 30-year-old woman presents with shortness of breath and bloody sputum. Physical examination reveals pulmonary crackles and abdominal ascites. A chest X-ray shows bilateral pleural effusions and marked hyperinflation of the lungs. A CT scan of the chest discloses thin-walled, air-containing cysts in a diffuse symmetric pattern. A lung biopsy is shown in the image. The patient responds favorably to antiestrogen and antiprogesterone therapy. Which of the following is the most likely diagnosis? (A) Bronchiectasis (B) Histiocytosis X (C) Lymphangioleiomyomatosis (D) Tuberculosis (E) Wegener granulomatosis

C. Lymphangioleiomyomatosis. Lymphangioleiomyomatosis is a rare interstitial lung disease that occurs in women of childbearing age. It is characterized by the widespread abnormal proliferation of smooth muscle in the lung (see photomicrograph), mediastinal and retroperitoneal lymph nodes, and the major lymphatic ducts. On gross examination, the lungs show bilateral, diffuse enlargement, with extensive cystic changes resembling those of emphysema. Hormonal ablation through oophorectomy and antiestrogen and progesterone therapy has shown some promise. None of the other choices exhibit this morphologic pattern. Diagnosis: Lymphangioleiomyomatosis

A 19-year-old man has a history of recurrent mucoid rhinorrhea with chronic sinusitis and otitis media since child- hood. He has experienced multiple bouts of pneumonia. His temperature is 37.7° C. On examination of his chest, there is tactile fremitus, rhonchi, and rales in lower lung fields. Nasal polyps are noted. A chest radiograph shows bronchial dilation with bronchial wall thickening, focal atelectasis, and areas of hyperinflation; his heart shadow appears mainly on the right. Which of the following abnormalities is he most likely to have? A α1-Antitrypsin deficiency B Atopy C Chloride ion channel dysfunction D Ciliary dyskinesia E HIV infection

D Ciliary dyskinesia He has Kartagener syndrome (sinusitis, bronchiectasis, and situs inversus associated with ciliary dyskinesia). There is an abnormality of ciliary dynein arms that diminishes the mucociliary function of the respiratory epithelium, predisposing to recurrent and chronic infections of both upper and lower respiratory tract. Bronchiectasis is ongoing destruction and dilation of bronchi with infection and airway obstruction. α1-Antitrypsin deficiency leads to pan lobular emphysema, mainly of lower lobes, and the upper respiratory tract is not involved. Atopy may be associated with nasal polyps, but leads to asthma, not bronchiectasis. Cystic fibrosis with CFTR gene mutations involving chloride ion channels can lead to widespread bronchiectasis, but generally not upper airway problems, and not situs inversus. HIV infection is marked by opportunistic infections with progression to AIDS, but usually without bronchiectasis, and no situs inversus.

A 59-year-old man who has smoked one pack of ciga- rettes per day for the past 43 years has developed a severe cough with hemoptysis over the past month. He has expe- rienced a 10-kg weight loss over the past year. On physical examination, he is afebrile. Laboratory studies show a serum Na+ of 120 mmol/L; K+, 3.8 mmol/L; Cl-, 90 mmol/L; CO2, 24 mmol/L; glucose, 75 mg/dL; creatinine, 1.2 mg/dL; calcium, 8.1 mg/dL; phosphorus, 2.9 mg/dL; and albumin, 4.2 g/dL. Which of the following findings is most likely to be seen on a chest radiograph? A Bilateral upper lobe cavitation B Diaphragmatic pleural calcified plaques C Extensive areas of infiltrates D Invasive perihilar mass E Pneumothorax F Subpleural nodule with hilar adenopathy G Upper lung nodule with air-fluid level

D Invasive perihilar mass The patient probably has a small cell anaplastic (oat cell) carcinoma of the lung, which is most likely to produce a paraneoplastic syndrome with the syndrome of inappropriate secretion of antidiuretic hormone (SIADH), marked by free water retention with hyponatremia. Oat cell cancers tend to be central masses, and they are strongly associated with smoking. Upper lobe cavitation suggests secondary tuberculosis. Diaphragmatic pleural plaques can be a feature of pneumoconioses, particularly asbestosis. Infiltrates can suggest an inflammatory process. Pneumothorax is most likely to occur from chest trauma, not from a neoplasm. A sub pleural nodule with hilar adenopathy is the classic Ghon complex of primary tuberculosis, which is unlikely to manifest with hemoptysis. An air-fluid level suggests liquefaction in an abscess.

A 68-year-old man has had worsening dyspnea and orthopnea for the past 3 years with increased production of frothy sputum. On examination, crackles are auscultated at lung bases. A chest radiograph shows bilateral interstitial infiltrates, distinct Kerley B lines, and a prominent left heart border. Laboratory studies show Na+, 135 mmol/L; K+, 3.8 mmol/L; Cl-, 99 mmol/L; CO2, 25 mmol/L; glucose, 76 mg/ dL; creatinine, 1.5 mg/dL; and urea nitrogen, 30 mg/dL. Frac- tional excretion of sodium is less than 1%. Plasma renin, al- dosterone, and antidiuretic hormone levels all are increased. B-type natriuretic peptide (BNP) is 200 pg/mL (normal <100 pg/mL). Which of the following pathologic findings is this man most likely to have? A Aldosteronoma B Bilateral adrenal atrophy C Chronic glomerulonephritis D Ischemic heart disease E Pulmonary fibrosis F Small cell carcinoma

D Ischemic heart disease He has left-sided heart failure with pulmonary edema and congestion. His reduced cardiac output leads to diminished renal blood flow that stimulates the renin-angiotensin mechanism to retain salt and water to increase plasma volume. He has prerenal azotemia with a high BUN-to-creatinine ratio and low fractional excretion of sodium. The other options do not explain his pulmonary edema. An aldosterone secreting adrenal adenoma (Conn syndrome) would increase aldosterone, but decrease the plasma renin. In chronic adrenal failure (Addison disease), there should be hyperkalemia and hypoglycemia accompanying hyponatremia. Chronic glomerulonephritis with chronic renal failure would be associated with a BUN-to-creatinine ratio around 10:1. Pulmonary fibrosis would lead to cor pulmonale and a prominent right heart border with features of right-sided congestive heart failure. The syndrome of inappropriate antidiuretic hormone (ADH) is a paraneoplastic syndrome that can occur with pulmonary small cell carcinomas, and secretion of antidiuretic hormone (SIADH) would increase ADH and cause more severe hyponatremia, but would not have a major effect on the renin-angiotensin mechanism, and sodium excretion would be higher.

A study of HIV-infected persons shows that those with CD4+ lymphocyte counts below 100 cells/μL are found to be at increased risk for pulmonary infections. Some of them have concurrent hepatosplenomegaly and lymphadenopathy, as well as malabsorption with weight loss, night sweats, and fever. Bronchoalveolar lavage specimens examined micro- scopically show macrophages filled with acid-fast infectious organisms. Which of the following infections have these per- sons developed? A Aspergillus niger B Candida albicans C Legionella pneumophila D Mycobacterium avium-complex E Nocardia asteroides F Pseudomonas aeruginosa

D Mycobacterium avium-complex Nontuberculous mycobacterial infections such as Mycobacterium avium-complex (MAC) are likely to become disseminated illnesses in immunocompromised persons. In immunocompetent persons, such infections are more likely to resemble tuberculosis. The acid-fast MAC organisms proliferate profusely in macrophages within the mononuclear phagocyte system. Extensive and severe aspergillosis and candidiasis are more likely to occur with profound neutropenia, not lymphocytopenia. Legionella produces an extensive bacterial pneumonia, not disseminated disease. Nocardiosis occurs in immunocompromised patients, and the organisms can be weakly acid-fast, but focal nodules or a chronic abscessing inflammatory response are more likely.

A 56-year-old man has had fever, night sweats, and a 3-kg weight loss over the past 4 months. In the past month, he has had episodes of hemoptysis. On physical examination there are upper lobe rales. He has hypoxemia. The appearance of his chest radiograph is shown in the figure. He is most likely to have an infection with which of the following organisms? A Candida albicans B Influenza A C Legionella pneumophila D Mycobacterium tuberculosis E Mycoplasma pneumoniae F Nocardia asteroides

D Mycobacterium tuberculosis The radiograph in the figure shows prominent upper lobe cavitation, typical of reactivation-reinfection tuberculosis in adults. Candida is a rare cause of lung infection. Influenza viral infections have mainly interstitial mononuclear inflammation. Bacterial organisms such as Legionella are more likely to produce a widespread bronchopneumonia with alveolar neutrophilic exudates. Mycoplasma infection produces mainly interstitial mononuclear inflammation. Nocardiosis of the lung appears mainly as chronic abscessing inflammation.

A 51-year-old man received an orthotopic cardiac transplant a month ago. He has developed a fever with cough over the past 5 days. On physical examination his temperature is 37.5°C. A chest CT scan shows consolidation with abscess formation involving the left lower lobe. A sputum gram stain reveals normal upper respiratory tract flora. He does not respond to antibiotic therapy over the next 6 months. His mental status deteriorates and MR imaging of the brain shows multiple abscesses. He is most likely to have an infection with which of the following organisms? A Mycoplasma pneumoniae B Aspergillus fumigatus C Mycobacterium avium-complex D Nocardia braziliensis E Cytomegalovirus F Pneumocystis jiroveci

D Nocardia braziliensis Nocardia braziliensis infection can persist and lead to chronic abscesses. It can complicate the course of immunocompromised patients. Though the lung is the portal of entry, dissemination to brain is common. A - This produces a primary atypical pneumonia without abscess or alveolar filling. B - Aspergillus does not typically produce abscesses. C - MAI typically affects AIDS patients, and lung involvement is minimal. E - CMV does not produce abscesses. F - Pneumocystis pneumonia is rarely disseminated and most often produces a diffuse pneumonic consolidation with minimal inflammatory cell infiltration and no abscess formation.

A study is performed reviewing medical records of adults presenting with sudden onset of severe dyspnea. They were afebrile, with absent breath sounds over an entire lung, and chest x-ray showing pulmonary atelectasis of an entire lung. Which of the following conditions is most likely to produce these findings? A Aspiration of a foreign body B Pulmonary embolism C Squamous cell carcinoma D Penetrating chest trauma E Bronchiectasis

D Penetrating chest trauma Penetrating chest trauma would lead to pneumothorax with lung collapse (atelectasis). B - Occlusion of pulmonary arterial circulation does not collapse a lung.

A 35-year-old woman has experienced multiple bouts of severe necrotizing pneumonia since childhood, with Hae- mophilus influenzae, Staphylococcus aureus, Pseudomonas aerugi- nosa, and Serratia marcescens cultured from her sputum. She now has a cough productive of large amounts of purulent sputum. On physical examination, there is dullness to percus- sion with decreased breath sounds over the right mid to lower lung fields. A chest radiograph shows areas of right lower lobe consolidation. A bronchogram shows marked dilation of right lower lobe bronchi. Which of the following mechanisms is the most likely cause of her disease? A Congenital malformation of the bronchial walls B Damage to bronchial mucosa by major basic protein of eosinophils C Diffuse infiltration by bronchogenic carcinoma D Recurrent inflammation with bronchial wall destruction E Unopposed action of neutrophil-derived elastase on bronchi

D Recurrent inflammation with bronchial wall destruction Bronchiectasis is a chronic obstructive airway disease from irreversible dilation of bronchi that results from inflammation and destruction of bronchial walls after prolonged infections or obstruction. Serious bouts of pneumonia can predispose to bronchiectasis. Congenital chondromalacia weakening the bronchial wall is rare. Bronchial mucosal damage by eosinophils occurs in bronchial asthma. It does not cause destruction of the bronchial wall. Bronchioloalveolar carcinoma may mimic an infiltrative pneumonia because of its lepidic pattern of spread, but it mainly produces a mass effect, and it does not start in childhood. Unopposed action of elastases damages the elastic tissue of alveoli, giving rise to emphysema.

A study is conducted of individuals who smoked at least one pack of cigarettes per day for 30 years. These individu- als undergo pulmonary function testing, and a large subset is found to have decreased FEV1, normal to decreased FVC, and FEV1/FVC ratios less than 70%. Autopsy data from this sub- set of individuals in the study with a low FEV1/FVC ratio are analyzed. Which of the following respiratory tract structures in the lungs is likely to be affected most by the underlying disease? A Alveolar duct B Alveolar sac C Bronchi D Respiratory bronchiole E Terminal bronchiole

D Respiratory bronchiole Centrilobular emphysema results from damage to the central part of the lung acinus, with dilation that primarily affects the respiratory bronchioles. There is relative sparing of the distal alveolar ducts and alveolar sacs. Bronchi have cartilage that is not affected by emphysema. In panacinar emphysema, the lung lobule is involved from the respiratory bronchiole to the terminal alveoli. In paraseptal emphysema, the distal acinus is involved.

A 42-year-old man has had chronic sinusitis for 7 months. He now has malaise and a mild fever that have persisted for 3 weeks. On physical examination, his temperature is 37.9° C. On auscultation, a few crackles are heard over the lungs. Labora- tory studies show serum urea nitrogen of 35 mg/dL; creatinine, 4.3 mg/dL; ALT, 167 U/L; AST, 154 U/L; and total bilirubin, 1.1 mg/dL. A chest radiograph shows scattered 1-cm pulmo- nary nodules. A transbronchial lung biopsy is performed, and microscopic examination shows necrotizing granulomatous capillaritis, a poorly formed granuloma, and intra-alveolar hemorrhage. Urinalysis shows RBCs with RBC casts. Autoanti- bodies to which of the following are most likely to be present in this man? A DNA topoisomerase I (Scl-70) B Double-stranded DNA (dsDNA) C Glomerular basement membrane (GBM) D Serine proteinase 3 (PR3-ANCA) E Thermoactinomyces vulgaris

D Serine proteinase 3 (PR3-ANCA) Polyangiitis with granulomatosis, a form of ANCA- associated vasculitis, may affect multiple organs, but the lung and kidney are most often involved. The C-ANCA test (PR3) result is often positive in granulomatosis with polyangiitis, whereas a positive P-ANCA (MPO) result suggests microscopic polyangiitis. Renal and pulmonary disease may be present in Goodpasture syndrome; there may be a positive result for anti-glomerular basement membrane antibody, but no C-ANCA or P-ANCA positivity. Of the collagen vascular diseases, systemic sclerosis is more likely to produce significant pulmonary disease, but hemoptysis is not a prominent feature, and the C-ANCA result is unlikely to be positive. In systemic lupus erythematosus, renal disease is far more likely than pulmonary disease, and C-ANCA or P-ANCA positivity is not expected, but anti-Smith and anti-dsDNA are most specific for SLE. Anti-GBM is a feature of Goodpasture syndrome with extensive intra-alveolar hemorrhage. In hypersensitivity pneumonitis, often the result of inhalation of thermophilic actinomycetes, an initial type III hypersensitivity response is followed by a type IV response, and renal disease is not expected.

A 59-year-old man has been a cigarette smoker for the past 42 years. He has noted some blood-streaked sputum on coughing during the past week. He also has back pain. A chest radiograph shows a small 3 cm right hilar mass with several 1 to 2 cm peripheral lung nodules. A bone scan reveals multiple areas of increased uptake in the vertebrae, ribs, and pelvis. A sputum cytology reveals the presence of clusters of small cells having hyperchromatic nuclei and almost no cytoplasm. Which of the following laboratory test findings is he most likely to have as a consequence of his lung disease? A Positive antinuclear antibody B Platelet count of 55,000/microliter C Plasma cortisol at 8 am of 5 microgm/dL D Serum sodium of 113 mmol/L E Serum uric acid of 14 mg/dL

D Serum sodium of 113 mmol/L He has a small cell anaplastic carcinoma with widespread metastases. The syndrome of inappropriate ADH producing hyponatremia is one form of paraneoplastic syndrome seen with this particular carcinoma. A - Paraneoplastic syndromes are not generally associated with autoimmune diseases or markers. B - Thrombocytopenia is not a feature of a paraneoplastic syndrome. Hypercoaguability (Trousseau syndrome) is a feature of malignancy, most often adenocarcinoma. C - One form of paraneoplastic syndrome is Cushing syndrome from ectopic ACTH production, which should elevate the serum cortisol. The cortisol value listed here is not high. E - The turnover rate of lung cancers is generally not high enough to elevate the uric acid.

A 28-year-old male present with cough and hemoptysis. He has had previous episodes of sinusitis. A transbronchial biopsy is performed and the diagnosis of polyagniitis with granulomatosis is made. Laboratory and histologic features include: A. Anti GBM antibodies and necrotizing granulomas B. Specific HLA subtypes and immune deposits along the basement membranes C. Crescentic glomerulosclerosis and linear immunoglobulin deposits D. PR3-ANCAs and necrotizing vasculitis E. PR3-ANCAs and caseating granulomas

D. PR3-ANCAs and necrotizing vasculitis

A 47-year-old man has lost 6 kg in 5 months. He has had a cough with hemoptysis along with pleuritic chest pain for the past 2 weeks. On physical examination his temperature is 37.5°C. A chest x-ray reveals a bilateral and predominantly upper lobe reticulonodular pattern of infiltrates with cavitation. A sputum sample is obtained and on light microscopic examination shows epithelioid cells with necrotic debris. Laboratory studies show a WBC count of 5890/microliter with 78% granulocytes, 15% lymphocytes, and 7% monocytes. Which of the following additional histologic findings is most likely to be present in his sputum? A Branching, septated hyphae B Pleomorphic cells with dark, angular nuclei C Clusters of small RBC-sized cysts staining with GMS D Large cells with intranuclear inclusions E Acid fast bacilli

E Acid fast bacilli The hemoptysis suggests that the granulomas have eroded enough parenchyma and involved a bronchus. A granulomatous reaction is typical for Mycobacterium tuberculosis infection. A - Aspergillus infection is not common. It is more likely to produce a fungus ball. Persons who are neutropenic are at greatest risk. The inflammatory reaction could vary from acute to mixed to granulomatous. B - A squamous cell carcinoma is usually a central mass lesion. It is unlikely that epithelioid cells would appear on biopsy. C - PCP in rare cases has a granulomatous pattern, but the granulomas would be small and unlikely to lead to hemoptysis. D - Intranuclear inclusions suggest infection with a herpesvirus, and the largest cell enlargement occurs with cytomegalovirus. They are unlikely to be found along with epithelioid cells.

Following an acute pharyngitis lasting 4 days, a 10-year-old boy develops neck pain and marked halitosis. On physical examination is breath is very malodorous. A CT scan shows an abscess in the peritonsillar region. Laboratory studies include a culture of the abscess which grows anaerobic flora. Which of the following aerobic organisms is most likely to be cultured from his abscess? A Staphylococcus aureus B Hemophilus influenzae C Corynebacterium diphtheriae D Bordetella pertussis E Group A Streptococcus

E Group A Streptococcus A peritonsillar abscess is usually a complication of a 'strep' throat in a child. A - Staphylococcal infections of the pharynx are not common in children. B - Hemophilus more typically produces an epiglottitis that may be obstructive. It can also lead to pneumonia and to meningitis. C - Diptheria is rare nowadays due to immunization. In the acute disease there is a pseudomembrane that is formed, not an abscess. D - This is the causative agent for whooping cough, which is rare because of immunization. There is a laryngotracheobronchitis, but rarely mucosal erosion.

A 45-year-old woman, a nonsmoker, has had a chronic nonproductive cough for 6 months along with 8-kg weight loss. On physical examination there are no remarkable findings. Her chest radiograph shows a right peripheral sub-pleural mass. A fine-needle aspiration biopsy is performed, and she undergoes a right lower lobectomy. The microscopic examination of the lesion shows glands invading the surrounding lung. Which of the following molecular test findings is most useful in deciding if her cancer may benefit from therapy targeting a tyrosine kinase? A Amplification of FGFR1 gene B Inactivation of CDKN2A gene C Loss of both copies of TP53 D Mutation in K-RAS gene E Rearrangement of ALK gene

E Rearrangement of ALK gene The ALK gene encodes a receptor tyrosine kinase that gets activated by fusion with the EML gene. The fused ALKEML (just as BCR-ABL in CML) can be targeted by inhibitors of tyrosine kinases. Gain-of-function mutations in multiple genes encoding receptor tyrosine kinases, including EGFR, ALK, ROS, MET, and RET, are often mutated in adenocarcinomas, which tend to be peripheral masses. Along with EGFR, ALK gene mutations can be targeted in the subset of lung adenocarcinomas that bear these molecular changes. All the other genes, although more commonly mutated in a variety of cancers, cannot be targeted by any of the available drugs.

At autopsy, a 60-year-old man is found to have a peripheral 7 cm area of golden-yellow consolidation on sectioning of the left lung. Microscopically, this area has alveoli filled with foamy macrophages. Which of the following conditions involving his lung is most likely to be responsible for this finding? A Mycoplasma pneumoniae infection B Cystic fibrosis C Adenocarcinoma D Silicosis E Squamous cell carcinoma F Malignant mesothelioma G Thromboembolism

E Squamous cell carcinoma Most squamous cell carcinomas are located centrally and arise in bronchi, leading to obstruction of a large airway that can cause a distal lipid pneumonia. In this case, the breakdown of lung tissue distal to the mass yields an 'endogenous lipid pneumonia' with many macrophages. Some degree of atelectasis may also be present from air resorbtion distal to the obstruction. A - Mycoplasma infection produces an atypical pneumonia with interstitial infiltrates. Obstruction of airways is unlikely. B - The obstruction with inflammation of cystic fibrosis leads to bronchiectasis. C - Most adenocarcinomas are peripheral and do not arise within a large bronchus and thus are less likely to produce airway obstruction. D - The silicotic nodules are usually small (a few millimeters) and do not obstruct bronchi. F - Malignant mesotheliomas arise in the pleura and are peripheral masses unlikely to obstruct proximal bronchi to produce a lipid pneumonia. G - Thromboemboli obstruct the pulmonary arterial system, not the bronchi.

A 51-year-old man has a history of chronic alcohol abuse. He is found in a stuporous condition after 3 days of binge drinking. On physical examination, his temperature is 39.2° C. A few crackles are heard on auscultation of the right lung base. A chest radiograph shows a 3-cm round lesion with an air-fluid level in the right lower lobe. Which pair of the fol- lowing organisms is most likely to be detected in his sputum? A Cryptococcus neoformans and Candida albicans B Cytomegalovirus and Pneumocystis jiroveci C Mycobacterium tuberculosis and Aspergillus fumigatus D Nocardia asteroides and Actinomyces israelii E Staphylococcus aureus and Bacteroides fragilis

E Staphylococcus aureus and Bacteroides fragilis A lung abscess often results from aspiration, which can occur in individuals with a depressed cough reflex or in neurologically impaired individuals (e.g., owing to acute alcoholism, anesthesia, or Alzheimer disease). Aspiration into the right lung and the lower lobe is more common because the main bronchus to the left lung is more acutely angled. Bacterial organisms are most likely to produce abscesses, and the infection may be polymicrobial. The most common pathogen is Staphylococcus aureus, but anaerobes such as Bacteroides, Peptococcus, and Fusobacterium spp. also may be implicated. These anaerobes normally are found in the oral cavity and are readily aspirated. The purulent, liquefied center of the abscess can produce the radiographic appearance of an air fluid level. Cytomegalovirus, Pneumocystis, and cryptococcal infections are seen in immunocompromised individuals and do not typically form abscesses. Candida pneumonia is rare. Nocardial and actinomycotic infections often lead to chronic abscesses without significant liquefaction and affect immunocompromised individuals. Tuberculosis can produce granulomatous lesions with central cavitation that may be colonized by Aspergillus, although not over a few days.

A 64-year-old woman who has suffered shock and sepsis experiences declining respiratory function and is placed on a ventilator. Despite intensive therapy, the patient dies 3 weeks later in respiratory failure. Histologic examination of the lungs at autopsy is shown in the image. Which of the following best describes the pathologic findings in this autopsy specimen? (A) Atelectasis (B) Bronchiectasis (C) Bronchopneumonia (D) Lobarpneumonia (E) Pulmonary fibrosis

E. Pulmonary fibrosis The photomicrograph shows hyaline membranes, thickening of the alveolar walls, and loose connective tissue. This organizing phase of diffuse alveolar damage begins about 1 week after the initial injury. This phase is marked by the proliferation of fibroblasts within the alveolar walls. Alveolar macrophages digest the remnants of hyaline membranes and other cellular debris. Loose fibrosis then thickens the alveolar septa. This fibrosis resolves in mild cases, but in severe cases, it may progress to restructuring of the pulmonary parenchyma and cyst formation. The photograph does not display features of the other choices. Diagnosis: Diffuse alveolar damage

A 63-year-old man with small cell carcinoma of the left mainstem bronchus begins chemotherapy. During the treat- ment period, he becomes febrile and develops a productive cough. The temperature is 38.7°C (103°F), respirations are 32 per minute, and blood pressure is 125/85mmHg. A CBC shows leukocytosis (WBC = 18,500/μL). The patient's cough worsens, and he begins expectorating large amounts of foul- smelling sputum. A chest X-ray shows a distinct cavity with an air/fluid level distal to the tumor area. Which of the following is the most likely diagnosis? (A) Atelectasis (B) Bronchiectasis (C) Ghoncomplex (D) Lobarpneumonia (E) Pulmonary abscess

E: Pulmonary abscess. Lung abscess is a localized accumulation of pus accompanied by the destruction of pulmonary parenchyma, including alveoli, airways, and blood vessels. The most common cause of pulmonary abscess is aspiration, often in the setting of depressed consciousness. Pulmonary abscess is also a common complication of lung cancer. Cystic abscess contains purulent exudates and is contained by a fibrous wall. Abscess cavities are often partially filled with pus and air, which accounts for the X-ray finding of an "air/fluid level." Infl ammation is usually present in the surrounding pulmonary parenchyma. Foul-smelling sputum may be expectorated if an abscess is connected to a bronchus. None of the other choices present as cavitary lesions. Diagnosis: Pulmonary abscess, small cell carcinoma of lung

A 4-year-old healthy girl from Utrecht in the Nether- lands has had a fever with dyspnea, tachypnea, nonproductive cough, myalgias, and rhinorrhea for 3 days. On auscultation of her chest there are inspiratory and expiratory wheezes. A chest radiograph shows bilateral diffuse interstitial infil- trates. She recovers in 2 weeks with no sequelae. Which of the following organisms is most likely to be identified by PCR of her respiratory secretions? A Group A Streptococcus B Bordetella pertussis C Candida albicans D Cytomegalovirus E Haemophilus influenzae F Human metapneumovirus

F Human metapneumovirus Human metapneumovirus is second only to respiratory syncytial virus as a cause for pediatric lower respiratory infection, and the two are indistinguishable clinically. She mainly has bronchiolitis. Like most viral infections, round cell infiltrates are interstitial, and those at greatest risk are the very young, elderly, and immunocompromised. Group A streptococcal infections typically produce pharyngitis. Pertussis is rare when childhood vaccinations are done; it causes whooping cough from upper airway involvement. Candida infections of the lower respiratory tract are uncommon even in immunocompromised persons. Cytomegalovirus is most likely to involve the lungs of immune compromised persons. H. influenzae is more likely to produce neutrophilic patchy infiltrates with productive cough.

A 24-year-old man has had increasing dyspnea for the past 10 weeks. On physical examination, he is afebrile. There is dullness to percussion over the lungs posteriorly and decreased breath sounds. A chest radiograph shows large bilateral pleu- ral effusions and widening of the mediastinum. Thoracentesis is performed on the left side and yields 500 mL of milky white fluid. Laboratory studies of the fluid show a high protein content; microscopy shows many lymphocytes and fat globules. What is the most likely cause for these findings? A Bacterial pneumonia with empyema B Congenital heart disease with congestive failure C Marfan syndrome with aortic dissection D Micronodular cirrhosis with hypoalbuminemia E Miliary tuberculosis with granulomatous pleuritis F Non-Hodgkin lymphoma with lymphatic obstruction

F Non-Hodgkin lymphoma with lymphatic obstruction The pleural fluid findings are typical of chylothorax, which is uncommon but distinctive. Lymph fluid is rich in lymphocytes, protein, and lipid (chylomicrons). Disruption of the thoracic duct in the posterior chest is most likely to cause chylothorax, and malignant neoplasms, such as a non-Hodgkin lymphoma, are most likely to do this. An empyema is composed of pus formed from neutrophilic exudation and would appear cloudy and yellow. Congenital heart disease can lead to congestive heart failure with a serous effusion. Aortic dissection is an acute condition that can produce a hemothorax. Cirrhosis is more likely to be associated with ascites or liver failure with hypoalbuminemia leading to hydrothorax. Miliary tuberculosis is seen as a reticulonodular pattern on a chest radiograph; tuberculosis may produce hemorrhagic effusions.

A 50-year-old man has developed truncal obesity, back pain, and skin that bruises easily over the past 5 months. On physical examination, he is afebrile, and his blood pressure is 160/95 mm Hg. A chest radiograph shows an ill-defined, 4-cm mass involving the left hilum of the lung. Cytologic ex- amination of bronchial washings from bronchoscopy shows round epithelial cells that have the appearance of lymphocytes but are larger. The patient is told that, although his disease is apparently localized to one side of the chest cavity, surgical treatment is unlikely to be curative. He also is advised to stop smoking. Which of the following neoplasms is most likely to be present in this patient? A Adenocarcinoma B Bronchial carcinoid C Bronchioloalveolar carcinoma D Large cell carcinoma E Non-Hodgkin lymphoma F Small cell anaplastic carcinoma G Squamous cell carcinoma

F Small cell anaplastic carcinoma Cushing syndrome is a paraneoplastic syndrome resulting from ectopic corticotrophin production (most often from a pulmonary small cell carcinoma), which drives the adrenal cortices to produce excess cortisol. Small cell carcinomas are aggressive neuroendocrine tumors that tend to metastasize early. Even when they appear to be small and localized, they are not or will not remain so. Surgery is not an option for these patients. They are treated as if they have systemic disease; some chemotherapy protocols afford benefit for 1 year or more, but cure is uncommon. Adenocarcinomas and large cell carcinomas tend to be peripheral neoplasms in the lung, and they are less likely to produce a paraneoplastic syndrome. Bronchial carcinoids at the more benign end of the neuroendocrine tumor spectrum tend to be small and are not likely to produce paraneoplastic effects; rarely, they produce carcinoid syndrome. Non-Hodgkin lymphomas rarely occur within the lung, are not associated with smoking, and do not produce Cushing syndrome. Squamous cell carcinomas can be central and occur in smokers, but they are more likely to produce hypercalcemia as a paraneoplastic syndrome.

A 47-year-old woman, a non-smoker, has a 4-month his- tory of mild but persistent right-sided chest pain. On physical examination, there are no remarkable findings. A chest radio- graph shows a pleural mass on the right side. No pleural effu- sions are seen. Chest CT scan shows a localized, circumscribed 3 × 7 cm mass attached to the visceral pleura; the lungs and chest wall appear normal. At thoracotomy, the mass is excised. On microscopic examination, the mass is composed of spin- dle cells resembling fibroblasts with abundant collagenous stroma. With immunohistochemical staining, the spindle cells mark for CD34, but are cytokeratin-negative. There has been no recurrence of the lesion. Which of the following is the most likely diagnosis? A Bronchioloalveolar carcinoma B Hamartoma C Hodgkin lymphoma, nodular sclerosis type D Malignant mesothelioma E Metastatic breast carcinoma F Solitary fibrous tumor

F Solitary fibrous tumor The solitary fibrous tumor, or localized mesothelioma, of pleura (or peritoneum) is a rare neoplasm that appears as a pedunculated mass. There is no relationship to asbestos exposure or other environmental pathogens. Many do not recur or metastasize following resection, but larger tumor size, higher mitotic count, and greater patient age (>55 years) increase the risk for metastasis. Bronchioloalveolar carcinomas are peripheral (but intraparenchymal) masses with atypical epithelial cells growing along the framework of the lung. A hamartoma is a peripheral intraparenchymal mass with a significant component of fibrous connective tissue and usually with cartilage present. Hodgkin lymphoma is more likely to involve lymph nodes in the mediastinum. A malignant mesothelioma forms a pleural mass that is not circumscribed; the cells are atypical and cytokeratin positive. Metastases are typically multiple and often produce bloody effusions.

A 54-year-old woman has had an increasingly severe cough productive of yellowish sputum for four days. On physical examination, her temperature is 38.9° C, and diffuse crackles are heard in the left lower lung posteriorly. Labo- ratory studies show a WBC count of 11,990/mm3 with 71% segmented neutrophils, 9% bands, 16% lymphocytes, and 4% monocytes. The representative gross appearance of the affect- ed lung is shown in the figure. Which of the following pathogens is most likely to be cultured from this patient's sputum? A Cryptococcus neoformans B Mycobacterium kansasii C Mycoplasma pneumoniae D Nocardia brasiliensis E Pneumocystis jiroveci F Streptococcus pneumoniae

F Streptococcus pneumoniae The productive cough suggests an alveolar exudate with neutrophils, and the course is compatible with an acute infection. Bacterial organisms should be suspected. Pneumococcus is the most likely agent to be cultured in individuals acquiring pneumonia outside of the hospital, and particularly when a lobar pneumonic pattern is present, as in this case. The primary atypical pneumonia of Mycoplasma does not usually produce purulent sputum, unless there is a secondary bacterial infection, which is a common complication of viral and Mycoplasma pneumonias. Cryptococcal and mycobacterial infections typically produce granulomatous disease. Nocardiosis also is seen in immunocompromised patients and produces chronic abscessing inflammation. Pneumocystis pneumonia is seen in immunocompromised patients and is unlikely to produce a lobar pattern of infection.

A 64-year-old man, who is a chain-smoker, has had a cough and a 5-kg weight loss over the past 3 months. Physi- cal examination shows clubbing of the fingers. He is afebrile. A chest radiograph shows no hilar adenopathy, but there is cavitation within a 3-cm lesion near the right hilum. Labora- tory studies show a serum calcium of 12.3 mg/dL, phospho- rus of 2.4 mg/dL, and albumin of 3.9 g/dL. Bronchoscopy shows a lesion almost occluding the right main bronchus. A surgical procedure with curative intent is attempted. Which of the following neoplasms is most likely to be present in this patient? A Adenocarcinoma in situ B Kaposi sarcoma C Large cell anaplastic carcinoma D Metastatic renal cell carcinoma E Non-Hodgkin lymphoma F Small cell anaplastic carcinoma G Squamous cell carcinoma

G Squamous cell carcinoma Of all lung cancers, squamous cell carcinoma is most likely to produce paraneoplastic hypercalcemia, and there is a strong association with smoking. These tumors also can undergo central necrosis—hence a cavity may form. Localized squamous cell carcinomas, in contrast to small cell carcinomas, may be cured by surgery. Adenocarcinomas and large cell carcinomas tend to produce peripheral masses and generally are less likely to be associated with paraneoplastic syndromes. Kaposi sarcoma involving visceral organs is most often seen in association with AIDS, and it is often multifocal. Renal cell carcinomas may be associated with hypercalcemia, but metastases usually appear as multiple masses (although of all metastatic tumors, renal cell carcinoma is most likely to produce solitary metastases). Non-Hodgkin lymphomas generally do not have paraneoplastic effects; they are uncommon in the lung and are not associated with smoking. Small cell carcinomas are never localized enough for curative surgery (they are usually detected at an advanced stage), although they often produce various paraneoplastic syndromes, but less likely hypercalcemia.

A 70-year-old woman at an extended care facility for the past two years has increasing inability to perform activities of daily living. She can no longer recognize family members. She is lethargic and spends most of her days in a wheelchair or in bed. She develops an acute febrile illness and is noted to be coughing up increasing quantities of yellowish sputum. Her temperature is 38°C. A chest x-ray shows infiltrates involving the left lower lobe. A sputum sample shows numerous neutrophils and gram-positive diplococci. Which of the following infectious agents is most likely to cause her pulmonary disease? A Pneumocystis jiroveci B Listeria monocytogenes C Cryptococcus neoformans D Mycobacterium tuberculosis E Legionella pneumophila F Staphylococcus aureus G Streptococcus pneumoniae H Influenza A virus

G Streptococcus pneumonia She has a lobar pneumonia, which is most often a community aquired pneumonia (the nursing home counts as a community venue), following a debilitating course of Alzheimer disease. The most common organism is Streptococcus pneumoniae (pneumococcus). S. aureus is more likely to be a nosocomial infection and more likely to produce a bronchopneumonia with patchy infiltrates. Pneumocystis jiroveci (carinii) pneumonia is most often seen in immunocompromised patients and is typically bilateral and widespread in the lungs. Cryptococcal infections typically occur more frequently in immunocompromised persons. Tuberculosis produces a granulomatous pattern of infection. In an adult, reinfection or reactivation is the most likely pattern, with upper lobe involvement. Legionella can produce a florid pneumonia, typically in all lobes, but it is not common. Viral pneumonias are common in the elderly but produce interstitial pneumonitis with non-productive cough. However, viral pneumonias are often complicated by bacterial pneumonia.


Ensembles d'études connexes

MKT Research and Analysis- Ch. 9

View Set

chap 4, Chapter 1, Chapter 2, Chap 3, Chap 5

View Set

ob. Diagnostic Sonography TEST 2 Review

View Set

Adult Health 3 - exam 3 cranial nerves

View Set

Gov 15.1 SWYK 1 - Indiana's Only President

View Set